You are on page 1of 65

1

Indian Geography-Questions DPP 02

Q1. Consider the following pairs: b) 2 only


1. Indira col : Andaman and Nicobar c) Both 1 and 2
Island d) Neither 1 nor 2
2. Kibithu : Nagaland
3. Guhar Moti : Gujarat Q4. Consider the following statements:
Which of the above pairs is/are correctly matched? 1. Molassis basin is an important feature in the state
of Mizoram.
a) 1 only
2. Mahabharat range is located between the Indo-
b) 1 and 2 only
Nepal border.
c) 1 and 3 only
3. Aksai Chin forms a part of the Karakoram range.
d) 3 only
4. Nubra valley is accessed through the Khardung La
pass.
Q2. Consider the following peaks:
Select the correct answer using the codes given
1. Saramati below:
2. Nokrek a) 1 and 2 only
3. Makalu b) 2 and 4 only
4. Annapurna c) 1 and 3 only
5. Kamet d) 1 and 4 only
6. Nanda Devi

Which of the following is the correct sequence of Q5. Consider the following statements:
the above peaks in the east to west direction?
1. Bhabar areas are more suitable for crop cultivation
a) 2-1-4-3-5-6 than Terai.
b) 1-2-3-4-6-5 2. Khadar sediments are deposited near the flood
c) 2-1-4-3-6-5 plains.
d) 1-2-4-3-6-5 Which of the statements given above is/are
correct?

Q3. Consider the following statements: a) 1 only


1. The Indian Standard Meridian (ISD) cuts across b) 2 only
the centre of Andhra Pradesh. c) Both 1 and 2
2. The ISD pass through the state of Maharashtra. d) Neither 1 nor 2
Select the correct answer using the codes given
below:
a) 1 only
2

Q6. Consider the following statements:


1. All the rivers of the Central Highlands originate in Q9. Which of the following statements is most
the Vindhya and Satpura ranges. appropriate regarding the Islands of India?
2. Malda fault exists between the Malwa Plateau and a) The entire Island of Lakshadweep lies below the
the Shillong Plateau. 12-degree channel.
Which of the statements given above is/are b) Duncan Passage is situated between Swaraj dweep
incorrect? and the Shaheed dweep.
a) 1 only c) Agatti Island is situated to the north of the capital
b) 2 only Kavaratti.
c) Both 1 and 2 d) The highest peak of Andaman, Saddle Peak is
located in the South Andaman.
d) Neither 1 nor 2

Q10. Consider the following statements regarding


Q7. Consider the following ranges/mountains:
Lakshadweep:
1. Harishchandra
1. Adroit is the largest group of Islands in
2. Nallamala Lakshadweep.
3. Dandakaranya 2. Minicoy Island is separated from the rest of the
4. Ramgarh island through a 9-degree channel.
5. Malya giri Which of the statements given above is/are
6. Garhjat correct?
Arrange the above ranges in the south-north a) 1 only
direction: b) 2 only
a) 2-3-1-6-5-4 c) Both 1 and 2
b) 2-1-3-4-6-5 d) Neither 1 nor 2
c) 2-1-3-5-6-4
d) 1-2-4-3-6-5 Q11. Consider the following events:
1. Development of Malda gap
Q8. Consider the following statements regarding the 2. Upliftment of Potwar Plateau
Indian desert:
3. Upliftment of western Himalayas
1. Absence of marine deposits indicates that the 4. Emergent Western coasts
region was a part of the Peninsular plateau.
Which of the above events are responsible for the
2. This area shows opposing slopes in the Northern
evolution of present-day Himalayan rivers?
and the Southern parts.
a) 1, 2 and 4 only
Which of the statements given above is/are
b) 1, 2 and 3 only
correct?
c) 2, 3, and 4 only
a) 1 only
d) 1, 3, and 4 only
b) 2 only
c) Both 1 and 2
d) Neither 1 nor 2
3

Q12. Consider the following pairs: d) 1, 4, and 5


Rivers Source
1. Alaknanda Satopanth Glacier Q15. which of the following statement is correct?
2. Yamuna Bandarpunch a) Majority of Indian rivers flow through a rift valley.
3. Son Rajmahal hills b) Lacustrine deposits have been found from the
4. Kosi Dhaulagiri Shivalik.
Which of the above pairs are correctly matched? c) There is a sharp tilt of the Indian peninsula from
a) 1 and 4 only west to east.
b) 2 and 3 only d) Most of Tapi’s drainage basin lies in the state of
Gujarat.
c) 1 and 2 only
d) 2 and 4 only
Q16. With reference to river Tapi, consider the
following statements:
Q13. Which of the following National Parks are in the
1. It originates from the Amarkantak plateau.
vicinity of river Brahmaputra?
2. Purna and Panjhra are important tributaries of river
1. Kaziranga National Park
Tapi.
2. Pobitora wildlife Sanctuary
3. It flows through a rift valley.
3. Nameri National Park
4. It flows exclusively in Madhya Pradesh and
Select the correct answer using the codes given Gujarat.
below:
Which of the statements given above are correct?
a) 1 and 2 only a) 1 and 2 only
b) 2 and 3 only b) 1, 2 and 4 only
c) 1 only c) 2 and 3 only
d) 2 only d) 2, 3 and 4 only

Q14. Consider the following rivers: Q17. Which of the following is/are the left tributaries of
1. Raidak Indus River ?
2. Rind 1. Shyok
3. Sengar 2. Zaskar
4. Sindh 3. Chenab
5. Son Select the correct answer using the code given
Which of the above is/are the tributaries of below:
Yamuna? a) 1 and 2 only
a) 1,2, and 4 b) 2 and 3 only
b) 2, 3 and 4 c) 1 and 3 only
c) 3, 4, and 5 d) All the above
4

Q18. Arrange the following left bank tributaries of Q21. A relief which divides two areas drained by the
Ganga from west to east: river is known as:
1. Gandak a) Watershed
2. Gomti b) River basin
3. Ramganga c) Drainage pattern
4. Ghaghara d) Water management
Select the correct answer using the code given
below:
Q22. Which of the following are projects on river
a) 2-3-1-4 Chenab?
b) 2-4-1-3 1. Baglihar
c) 3-2-4-1 2. Dulhasti
d) 3-4-2-1 3. Salar
4. Tulbul
Q19. Which of the following are the characteristics of Select the correct answer from the given codes
the Peninsular Drainage System? below:
1. Senile topography a) 1 and 2 only
2. Antecedent drainage b) 2 and 3 only
3. Flow through shallow valleys c) 1, 2 and 3 only
Select the correct answer using the code given d) 1, 2 and 4 only
below:
a) 1 and 3 only
Q23. With reference to the places and the confluence of
b) 1 and 2 only rivers, which of the following pairs is incorrect?
c) 2 and 3 only a) Devprayag : Alaknanda - Bhagirathi
d) 1, 2 and 3 b) Rudraprayag: Alaknanda- Mandakini
c) Vishnuprayag:Alakhanada -Dauliganga
Q20. Which of the following is/are the reasons for lack d) Karnaprayag:Alakhananda- Nandan
of formation of delta by peninsular rivers falling
into the Arabian sea?
Q24. Recently the Cabinet has given nod to the first
1. Presence of Hard rocks.
project under the National Perspective Plan for
2. Presence of Steep slopes. interlinking of rivers. It envisages transferring
3. Flowing through faults. water from -
Select the correct answer from the given codes a) Cauvery to Tungabhadra
below: b) Godavari to krishna
a) 1 only c) Ken to betwa
b) 1, 2 and 3 d) Narmada to tapi
c) 3 only
d) 1 and 3 only
5

Q25. Which of the following rivers drain into Black sea? 4. SST gradient between the North Atlantic and the
1. Dnieper North Pacific Oceans
2. Volga Choose the Correct answer using the code given
3. Danube below:

Select the correct action from given codes below: a) 1,2 and 4 only
a) 1 only b) 2,3 and 4 only
b) 1, 2 and 3 c) 1 and 3 only
c) 3 only d) All of the above
d) 1 and 3 only
Q29. Which of the following are characteristic features
of the Indian Monsoons?
Q26. The ganga river is an example of:
1. Sudden Burst
a) Antecedent Drainage system
2. Inter season variability
b) Consequent Drainage System
3. Western Disturbances
c) Superimposed Drainage System
4. Intra season variability
d) Subsequent Drainage System
5. Sudden withdrawal
Choose the correct answer using the code given
Q27. Which of the following would have happened if the
Himalayas did not exist? below:

a) There would be no North Eastern Monsoon season a) 1,2 and 4 only


experienced in India. b) 3,4 and 5 only
b) The onset of monsoon would have taken place in c) 1,2,3 and 4 only
the winter season. d) 1,2,3,4 and 5
c) The Indian subcontinent would have recorded a
lower Long Period Average of rainfall received.
Q30. With reference to Western disturbances, consider
d) The Thar desert would have received very high the following statements:
rainfall every year.
1. These are weak tropical cyclones that originate
over the Mediterranean Sea.
Q28. Which of the following conditions are taken into 2. Jet streams play an important role in driving these
account by the Earth System Science Organisation high-pressure systems into India.
(ESSO) and the India Meteorological Department
3. The rainfall that occurs due to these disturbances is
(IMD) while making forecasts about the monsoon
known to be beneficial for the Rabi crops.
in India?
Which of the above statements is/are incorrect?
1. Mean Sea Level pressure in east Asia
a) 1 only
2. The surface air temperature over northwest Europe
b) 2 only
3. Sea Surface Temperature (SST) of the equatorial
South Indian Ocean c) 1 and 2 only
d) 1,2 and 3
6

Q31. With reference to Madden Julian oscillations c) Andaman and Nicobar Islands
(MJO), consider the following statements:
d) Tripura
1. Unlike a standing pattern like the El Niño, the
Madden–Julian oscillation is a traveling pattern
that propagates eastward above the warm parts of Q34. Which of the following pairs is /are correctly
matched?
the Indian and Pacific oceans.
Pre-monsoon showers State
2. A MJO cycle can last anywhere between 120 to
150 days. 1. Mango shower - Kerala

3. A MJO can completely nullify the impact of El- 2. Nor wester - Bengal and

Nino on monsoon rains in India. Bihar

Which of the above statements is/are correct? 3. Loo - Punjab

a) 1 only Select the correct answer using the code given


below:
b) 1 and 2 only
c) 1 and 3 only a) 1 only

d) 2 and 3 only b) 1, 2 and 3


c) 3 only
d) 1 and 3 only
Q32. With reference to heatwaves in India, which of the
following statements is incorrect?
a) In General, heatwaves are defined with reference Q35. Which of the following statements is/are correct
to the normal maximum temperature of a particular regarding western cyclonic disturbances ?
region. 1. The western cyclonic disturbances are brought into
b) Most heat waves in India are formed over the India by the easterly jet stream.
Peninsular region especially on the leeward side of 2. An increase in the prevailing night temperature
the Western ghats. generally indicates an advance in the arrival of
c) Inflow of Western disturbances in Northern India these cyclone disturbances.
can influence the occurrence of heat waves in Select the correct answer using the code given
Central India. below.
d) A cyclone hitting the cost of Odisha can trigger a
a) 1 only
heatwave in Punjab.
b) 2 only
c) Both 1 and 2
Q33. According to the Indian Meteorological
d) Neither 1 nor 2
Department, in which of the following states/union
territories does the South West Monsoon arrive
first?
a) Kerala
b) Lakshadweep
7

Q36. Which of the following pairs is/are correct vis a vis 2. Laterite Soils are widely cut as bricks for house
Koeppen scheme for classification of climate? construction.
1. Tropical climates — where the mean 3. They are poor in organic matter, Nitrogen,
monthly temperature phosphorus and calcium.
throughout the year is
Which of the statements given above is/are
over 18°C.
correct?
2. Warm temperate — where mean
a) 1 and 2 only
Climates temperature of the
b) 2 and 3 only
Coldest month is
c) 1 and 3 only
between 15°C and minus
4°C d) All of the above

3. Ice climates — where mean temperature


of the warmest month is Q39. With reference to the red and yellow soils, consider
under 0°C to 10 °C the following statements:
Select the correct action from given codes below: 1. They develop on crystalline igneous rocks in areas
a) 1 only of high rainfall.
b) 1, 2 and 3 2. The reddish color of the soil is due to the diffusion
of Iron and when it occurs in a hydrated form, it
c) 3 only
looks yellow.
d) 1 and 3 only
Which of the statements given above is/are
incorrect?
Q37. Which of the following statements is/are correct
a) 1 only
regarding the monsoon?
b) 2 only
1. Regional variations in monsoon climate help in
growing various types of crops. c) Both 1 and 2

2. Sudden monsoon bursts create a problem of soil d) Neither 1 nor 2


erosion over large areas in India.
3. Agricultural prosperity of India depends very Q40. Its soils are generally clayey, deep and
much on timely and adequately distributed rainfall. impermeable. They swell and become sticky when
Select the correct action from given codes below: wet and shrink when dried. So, during the dry
season, these soils develop wide cracks. Thus, there
a) 1 only
occurs a kind of ‘self-ploughing’ and rich in lime,
b) 1, 2 and 3 iron, magnesia and alumina.
c) 3 only This is the best description of:
d) 1 and 3 only a) Laterite Soils
b) Black soil
Q38. Consider the following statements regarding c) Loamy soil
laterite soil: d) Arid soil
1. Laterite Soils are more suitable for cashew cuts.
8

Q41. These cool temperate western margins are under


the permanent influence of the Westerlies all
Q44. Plants such as Tendu, palas, amaltas, bel, khair,
round the year. Summers are never very warm
axlewood are the examples of which of the
here. The natural vegetation of this climatic type is
following vegetation?
a deciduous forest like oak, elm, ash, birch, beech,
poplar, and hornbeam. a) Tropical thorny forest

Which of the following climatic types is described b) Tropical moist deciduous forest
in the passage given above? c) Tropical Dry deciduous forest
a) British Type Climate. d) Semi Evergreen Forest
b) Mediterranean Climate
c) Steppe Climate Q45. Which of the following conditions has led to the
d) North American climate establishment of plantations in the equatorial
regions?
1. Favorable hot and wet climate.
Q42. Consider the following statements regarding
equatorial climate: 2. Nutrient-rich tropical soils.

1. The equatorial vegetation comprises evergreen 3. High temperature and rainfall


trees that yield tropical hardwood. 4. Less problem of tree diseases.
2. The equatorial regions are generally densely Select the correct answer using the code given
populated. below.
Which of the statements given above is/are a) 1, 2 and 4 only
correct? b) 1 and 3 only
a) 1 only c) 1, 2 and 3 only
b) 2 only d) 2, 3 and 4 only
c) Both 1 and 2
d) Neither 1 nor 2 Q46. Consider the following statements:
1. Around 70 percent of wetland comprises areas
Q43. With reference to Himalayan vegetation, consider under paddy cultivation.
the following statements: 2. Around 23 percent of the world’s mangrove forests
1. The Deciduous forests found in the foothills of the are found in India.
Himalayas. 3. Mangroves grow along the coasts in the salt
2. Deodar is an endemic species of the western part marshes, tidal creeks, mud flats.
of the Himalayan range. Which of the statements given above is/are
Which of the statements given above is/are correct?
correct? a) 1 and 2 only
a) 1 only b) 2 and 3 only
b) 2 only c) All of the above
c) Both 1 and 2 d) 1 and 3 only
d) Neither 1 nor 2
9

Q47. Consider the following states: 1. The Trees shed leaves their leaves annually during
1. Haryana the autumn season.

2. Rajasthan 2. In India, the total area covered by these forests is


much more than the area covered by Littoral and
3. Gujrat
Swamp Forests.
4. Andhra Pradesh
3. In India, both South West monsoon and NorthEast
Where will you naturally find “Thorny Scrub Monsoon influence their growth.
Forests”?
Choose the correct answer using the code given
a) 1, 2 and 3 only below:
b) 2 and 3 only
a) 1 and 2 only
c) 1 and 4 only
b) 3 only
d) All of the above
c) 2 and 3 only
d) 1 and 3 only
Q48. Consider the following National parks:
1. Campbell Bay National Park
Q51. Consider the following statements:
2. Gugamal National Park
1. Some of the states in India have both Tropical
3. Dandeli Anshi National Park Evergreen and Tropical Dry Deciduous forests.
4. Mukundara Hills National Park 2. Some of the states in India have both Tropical
In which of the above National parks will you find Evergreen and Mangrove Forests.
Ebony and Mahogany trees naturally? 3. Some of the states in India have both Tropical
a) 1 and 2 only Evergreen forests and Alpine Vegetation.
b) 1 and 3 only 4. Every State in India has at least some geographical
c) 2 and 4 only area under Scrub Forests.
d) 1,3 and 4 only Which of the above statements are correct?
a) 1 and 2 only

Q49. Which of the following methods is the most b) 3 and 4 only


appropriate for checking soil erosion on Coastal c) 1,2 and 3 only
and arid areas? d) All of the above
a) Contour Bunding
b) Shelterbelts
Q52. In the context of forest surveys, ‘Margalef index’,
c) Contour Ploughing and the ‘The Shannon Wiener Index ‘ are used for
d) Check Dams measuring the
a) Total biomass of a forest

Q50. With reference to Tropical Moist Deciduous b) Carbon footprint of a forest


Forests, which of the following statements is/are c) Albedo of a forest
correct ? d) Species diversity in a forest
10

Q53. What are the applications of allelopathy in Q55. With reference to direct seeding of rice (DSR),
agriculture? consider the following statements:
1. Cost of weedicides can be reduced as weed 1. Direct-seeded rice matures 7 to 10 days earlier
management is possible. than transplanted rice.
2. Management and control of all insects and pests is 2. There is greater crop-weed competition in DSR
a possibility. than that in transplanted rice.
3. Reduction in development of yellow rust disease in 3. The seed requirement for DSR is lower than that
wheat is possible. required in the transplantation method.
4. Increased resistance towards abiotic stresses is Which of the above statements is/are correct?
possible. a) 1 and 2 only
Select the correct answer using the code given b) 2 and 3 only
below: c) 1 and 3 only
a) 1, 2 and 3 only d) 1,2 and 3
b) 1, 3 and 4 only
c) 1, 2 and 4 only Q56. Which among the following are the uses of
d) 2, 3 and 4 only seaweeds?
1. Food for humans
Q54. In the context of Indian agriculture, what are the 2. Potential indicators of pollution
advantages of Foliar feeding of Horticulture 3. Used in treatment of Goiter
plants?
Select the correct answer using codes given below:
1. It has proven to be an excellent method of a) 1 only
supplying secondary nutrients such as calcium,
b) 1 and 2
magnesium, and sulfur to plants.
c) 2 and 3
2. It can avoid the problem of leaching of nutrients in
soil. d) All of the above
3. It gives far more control over concentration of the
fertilizer solution than that achieved under direct Q57. With reference to mixed farming, which of the
application of Fertilizers into the roots. following statements is/are correct?
4. The nutrients can be mixed with pesticides, 1. Mixed farming is characterized by low capital
thereby achieving a synergic effect and additional expenditure on agricultural inputs.
saving on application costs.
2. Fodder crops are an important component of mixed
Choose the correct answer using the code given farming.
below:
Select the correct answer using the code given
a) 1, 2 and 4 only below.
b) 2, 3 and 4 only a) 1 only
c) 1 and 2 only b) 2 only
d) 1, 2, 3 and 4 c) Both 1 and 2
d) Neither 1 nor 2
11

Q58. Consider the following statements: d) Neither 1 nor 2


1. The 'Sarpagandha' plant is a medicinal plant found
exclusively in eastern and western ghats. Q61. Consider the following pairs:
2. It is used for treating various central nervous Phenomena Associated layers
system disorders. 1. Moho Discontinuity: Mantle- core
Which of the statements given above is/are 2. Gutenberg’s Discontinuity: Crust- mantle
correct? 3. Asthenosphere: Mantle only
4. Lithosphere: Crust only
a) 1 only
Which of the above pairs is/are incorrectly
b) 2 only
matched?
c) Both 1 and 2 a) 1, 2 and 4 only
d) Neither 1 nor 2 b) 1, 2 and 3 only
c) 2, 3 and 4 only
d) None of the above
Q59. With reference to inter cropping, consider the
following statements:
1. There is competition between competent crop i.e., Q62. Which of the following are the characteristics
Base crop and Intercrop. features of Permaculture :

2. Successful intercropping gives higher equivalent 1. It can help degraded ecosystems recover health and
yield, higher cropping intensity. wilderness.
3. The essential features of the Alley cropping are the 2. It promotes organic agriculture and completely
hedgerows are cut back at planting. eliminates use of pesticides.
3. It can be applied in any ecosystem, no matter how
Which of the statements given above is/are
degraded.
correct?
4. It aims to minimize symbiotic and synergistic
a) 1 only
relationships between site components.
b) 2 only
Which of the statements given above is/ are
c) 2 and 3 only correct?
d) 1 and 3 only a) 1, 2 and 3 only
b) 1 and 2 only
Q60. Consider the following statements regarding tillage c) 2, 3 and 4 only
in India: d) 3 and 4 only
1. Tillage is beneficial to reduce runoff of water and
reduce soil erosion.
Q63. ‘Limonite’,‘Siderite’ and ‘Lodestone’, sometimes
2. Zero Tillage degrades organic matter content due mentioned in the context of mining in India, are
to high mineralisation of Soil. inferior ores of
Which of the statements given above is/are
a) Copper
correct?
b) Manganese
a) 1 only
c) Iron
b) 2 only
d) Bauxite
c) Both 1 and 2
12

Q64. Which of the following minerals are found and is 1750 kilometres long. It connects the
naturally in the State of Odisha? Bombay High with Northern India.”
1. Iron ore Which of the following pipelines in India is best
2. Manganese described by the above excerpt?
3. Chromium a) Salaya-Koyali-Mathura Pipeline
4. Bauxite b) Hajira-Bijapur-Jagdishpur Gas Pipeline
Choose the correct answer using the code given c) Jamnagar-Loni LPG Pipeline
below: d) Kandla-Bhatinda Pipeline
a) 1 and 2 only
b) 2,3 and 4 only Q68. To promote inland water transport in the country
c) 1,3 and 4 only as an economical, environment friendly
d) 1,2,3 and 4 supplementary mode of transport to rail and road,
111 inland waterways have been declared as
‘National Waterways’ in India. In this context
Q65. Consider the following Minerals: consider the following statements:
1. Manganese 1. The declaration of National waterways was made
2. Nickel via an executive order.
3. Chromite 2. National Waterway-4 traverses through more states
4. Dolomite than National Waterway-1.
5. Bauxite 3. Recently in January 2021, all waterways have
6. Mica become fully operational.
Which of the above statements is/are correct?
Which of the above minerals are metallic in
nature? a) 1 only

a) 1,2,4 and 5 only b) 2 only

b) 1,2,3 and 5 only c) 1,2 and 3

c) 2,3,4 and 6 only d) None of the Above

d) 1,3,4,5 and 6 only


Q69. Consider the following statements:

Q66. In which of the following regions has The Ministry 1. The distance between rails is one metre is known
of Petroleum and Natural Gas not found as Broad Gauge.
prospective shale gas reserves in India? 2. The distance between the rails is 0.762 metre or
0.610 metre, known as Narrow gauge.
a) Gangetic Basin
3. Narrow gauge is confined to hilly areas of the
b) Assam-Arakan Basin
country.
c) Cauvery Basin
Which of the statements given above is/are
d) Bombay High Field
correct?
a) 1 and 2 only
Q67. “This pipeline runs through the states of Gujarat,
b) 2 and 3 only
Madhya Pradesh and Uttar Pradesh. It has been
c) 1 and 3 only
constructed by the Gas Authority of India Limited
d) 1, 2 and 3
13

Q70. With the reference to types of hydro power 2. Section 144 is imposed in urgent cases that have
stations, consider the following statements: the potential to cause trouble or damage to human
1. Impoundment power stations use the run of river life or property.
facility channels a portion of river through canal 3. Curfew is different from section 144, as it orders
and activates generators to produce electricity. people to stay indoors for a specific period of time.
2. Diversion power station uses a dam to store river
Which of the statements given above is/are
water in a reservoir and generate electricity.
correct?
3. Pumped storage power stations at the time
a) 1 and 2 only
demanded pumped storage facilities store energy
b) 2 and 3 only
by pumping water from a lower reservoir to an
upper reservoir. c) 1, 2 and 3
d) 1 and 3 only
Which of the statements given above is/are
incorrect?
a) 1 only Q73. With reference to National Commission for
Scheduled Tribe, consider the following
b) 2 and 3 only
statements:
c) 1 and 2 only
1. Article 338A gives powers to the NCST to oversee
d) 1, 2 and 3 the implementation of various safeguards under
any law provided to STs.

Q71. According to the Constitution, which of the 2. The Constitution lays down that the composition
following fundamental rights are available to both and strength of shall be determined by the
foreigners and citizens? Parliament.
1. Equality before law and equal protection of laws. 3. NCST presents a report to the President, annually
2. Movement, residence and profession. and at such other times as the Commission may
3. Right to elementary education. deem fit.

4. Protection of language, script and their culture. Which of the statements given above is/are
Select the correct answer using codes given below: incorrect?

a) 1, 2 and 3 only a) 1 only


b) 1 and 3 only b) 2 and 3 only
c) 2, 3 and 4 only c) 2 only
d) 2 and 4 only d) 1 and 3 only

Q72. Consider the following statements regarding the


section 144 of Criminal Procedure Code of 1973:
1. The maximum punishment for violation of section
144 of CrPC is one year.
14

Q74. Consider the following statements regarding


principles of reasonable accommodations:
Q76. Recently Legal Aid Clinic was in the news is
1. The principal provides additional support to launched by:
persons with disabilities to facilitate their
a) World Health Organisation
participation.
b) National Commission for Women
2. Workers living with HIV and AIDS are not
c) Delhi State Legal Services Authority
included in categories of workers in the guideline
d) Both (B) and (C)
on promoting diversity and inclusion through
workplace adjustments.
3. Rights of People with Disabilities Act, 2016, Q77. Consider the following statements regarding the
define ‘discrimination’ in Section 2(h) includes Central Bureau of Investigation:
‘denial of reasonable accommodation’. 1. CBI can take suo motu cognizance in a matter of
corruption involving government officials and
Which of the statements given above is/are
incidents of violent crime.
correct?
2. The Central Bureau of Investigation functions
a) 1, 2 and 3
under the Department of Personnel, Ministry of
b) 2 and 3 only
Personnel, Pension and Public Grievances.
c) 1 and 3 only
Which of the statements given above is/are
d) 1 and 2 only correct?

a) 1 only
Q75. Consider the following statements regarding b) 2 only
Foreigners Tribunal:
c) Both 1 and 2
1. Foreigners Tribunal is a quasi- Judicial body
d) Neither 1 nor 2
formed under the Foreigners’ Act, 1946.
2. All States and Union Territories can set up
tribunals to decide whether a person staying Q78. Recently “National eVidhan Application (NeVA)”,
illegally in India is a foreigner or not. a Mission Mode Project for Digital Legislatures
was in the news is developed by:
3. Only the State administration can move the
a) Ministry of Legal affairs
Tribunal against a suspect.
b) Ministry of Statistics and Program Implementation
Which of the statements given above is/are
c) Ministry of Parliamentary Affairs
correct?
d) Ministry of Home affairs
a) 1 and 2 only
b) 1 and 3 only
c) 2 and 3 only
d) 1, 2 and 3
15

Q79. Consider the following statements regarding office Which of the statements given above is/are
of Governor: correct?
1. Governor is the constitutional head of the state, a) 1 only
bound by the advice of the council of ministers of b) 2 only
that state. c) Both 1 and 2
2. The Governor has Constitutional discretion to d) Neither 1 nor 2
invite the leader of the largest party to form the
government when no party is in majority.
3. In the Nabam Rebia judgement (2016) Supreme Q82. Recently, the Montreux Convention was seen in
Court held that the exercise of Governor’s the news related to:
discretion Article 163 is limited and his choice of a) Reduce the Methane pollution.
action should not be arbitrary or fanciful. b) Control the Transboundary Movements of
Which of the statements given above is/are Hazardous Wastes.
correct? c) legal framework for all marine and maritime
a) 1 and 2 only activities.
b) 1, 2 and 3 d) The Regime of the Strait.
c) 2 and 3 only
d) 1 and 3 only Q83. Recently, PM-DAKSH (Pradhan Mantri Dakshta
Aur Kushalta Sampann Hitgrahi) was in the news,
with the aim of:
Q80. Recently a demand for creating Indian Legal
Services is in the news. Which of the following are a) Digital learning portal for Persons with disabilities
the concerns with the current practice of b) To Procure forest minor produce
appointing outsiders as Secretary-Generals? c) Skill development training programme to targeted
1. Against the principle of separation of powers. groups.
2. Lack of knowledge of the functioning of the d) To Skill Self defence training for Women.
legislature.
3. Impact Legislatures' role of holding the executive Q84. Consider the following statements regarding
accountable. Unlawful Activities (Prevention) Act 1967 (UAPA):
Select the correct answer using codes given below:
1. Once the police charge an individual under the
a) 1 and 2 only UAPA, it becomes extremely difficult for bail to
b) 1, 2 and 3 be granted.
c) 2 and 3 only 2. Supreme courts held that courts must accept the
d) 1 and 3 only state’s case without examining its merits.
Which of the statements given above is/are
incorrect?
Q81. Consider the following statements regarding
Electronically Transmitted Postal Ballot System a) 1 only
(ETPBS): b) 2 only
1. In ETPBS postal ballots are sent by post to c) Both 1 and 2
registered service voters for voting. d) Neither 1 nor 2
2. An NRI can vote in the constituency in which
his/her place of residence is located.
16

Q85. Consider the following statements regarding 3. The theme for the 7th edition is "Terra Nova:
‘Scheduled Tribe’: Impassioned, Impatient, and Imperilled”.
1. The Constitution defines the criteria for Which of the statements given above is/are
recognition of Scheduled Tribes under article 366. correct?
2. Fifth schedule deals with the administration of the a) 1 and 2 only
tribal areas in Assam, Meghalaya, Tripura and
b) 2 and 3 only
Mizoram.
c) 1 and 3 only
3. Darlong community belongs to tribal communities
of Arunachal Pradesh. d) 1, 2 and 3 only

Which of the statements given above is/are


incorrect? Q88. Which of the following statements is/are correct
regarding Palm oil?
a) 1 and 3 only
1. Indonesia, the world’s biggest producer, exporter,
b) 1, 2 and 3
and consumer of palm oil.
c) 1 and 2 only
2. Andhra Pradesh is the largest producer of palm oil
d) 2 and 3 only in India.
3. China is the biggest importer of palm oil and India
Q86. Consider the following statements regarding to is second largest.
The World Tourism Organization (UNWTO): Select the correct answer using the code given
1. It is the specialised agency of the United Nations. below:
2. World tourism day is celebrated every year on a) 1 only
September 27 around the world. b) 2 and 3 only
3. India is the host country to observe World Tourism c) 1 and 2 only
Day 2019 chosen by The World Tourism
d) 1, 2 and 3 only
Organization.
Which of the statements given above is/are
correct? Q89. Consider the following statements regarding
Desert National Park:
a) 1 and 2 only
1. It is situated on the western border of India within
b) 2 and 3 only
Jaisalmer and Barmer in Rajasthan.
c) 1 and 3 only
2. It is a place where the Great Indian Bustard,
d) 1, 2 and 3 only Chinkara and Rohida are found naturally.
Which of the statements given above is/ are
Q87. With reference to the Raisina Dialogue, consider correct?
the following statements: a) 1 only
1. The Raisina Dialogue was held for the first time on b) 2 only
1 March 2016.
c) Both 1 and 2
2. It is an annual geo-political event, organised by the
d) Neither 1 nor 2
NITI Aayog and Observer Research Foundation.
17

Q90. Consider the following statements regarding Q93. With reference to the International
International Finance Corporation: telecommunication Union, Consider the following
1. It is a member of the World Bank Group and was statements:
established in 1956. 1. International Telecommunication Union is the
2. It aims to advance economic development by United Nations specialized agency for Information
investing in strictly for-profit and commercial and Communication Technologies.
projects that purport to reduce poverty. 2. ITU currently restricts its membership for OECD
3. Its goals are to increase sustainable agriculture and G-20 members and private members to
opportunities, improve healthcare and increase increase research and development in the area of
access to financing for microfinance. 5G technology.

Which of the statements given above is/are 3. ITU works for three sectors namely Radio
correct? communication, Telecommunication
standardization and Telecommunication
a) 1 and 3 only development.
b) 2 only
Which of the statements given above is/are
c) 1 and 2 correct?
d) 1, 2 and 3
a) 1 and 2 only
b) 1 and 3 only
Q91. Consider the following statements:
c) 1, 2 and 3
1. Chile occupies a long, narrow strip of land d) 2 and 3 only
between the Andes to the east and the Pacific
Ocean to the west.
Q94. Consider the following statements regarding
2. Chile shares land borders with Bolivia to the north,
United Nations Population Funds (UNFPA):
Peru to the north-east, Drake Passage to the east,
and Argentina in the far south. 1. United Nations Population Funds began operations
in 1969 as the United Nations Fund for Population
Which of the statements given above is/are
Activity.
correct?
2. The agency work includes developing national
a) 1 only population management against gender-based
b) 2 only violence, and assists countries in areas such as
c) Both 1 and 2 human rights, reproductive health, population and
d) Neither 1 nor 2 development.
3. It is working with Government of India since 1974
to provide family planning and health
Q92. It shares land borders with Latvia to the north, services.
Belarus to the east and south, Poland to the south,
and Kaliningrad Oblast of Russia to the southwest. Which of the statements given above is/are
correct?
It has a maritime border with Sweden to the west
on the Baltic Sea, this is the best description of: a) 1 and 2 only
a) Estonia b) 1, 2 and 3
b) Ukraine c) 2 and 3 only
c) Lithuania d) 1 and 3 only
d) Belarus
18

Q95. With reference to the SVANidhi se Samriddhi a) 1 only


Program, Consider the following statements: b) 2 and 3 only
1. ‘SVANidhi se Samriddhi’ program launched by c) 1 and 2 only
The Ministry of Labour and Employment. d) 1 and 3 only
2. Its purpose is to provide social security benefits to
street vendors for their holistic development and
Q98. Consider the following statements regarding the
socio-economic upliftment.
Sarmat Intercontinental Ballistic Missile:
3. Its implementing partner is the Quality Council of
India (QCI). 1. The Sarmat is a Russian liquid-fueled
intercontinental ballistic missile capable of
Which of the statements given above is/are nuclear charges.
correct?
2. Sarmat is designed to elude anti-missile defense
a) 3 only systems.
b) 2 and 3 only Which of the statements given above is/are
c) 1 and 2 only correct?
d) 1 ,2 and 3 a) 1 only
b) 2 only
Q96. regarding the International Atomic Energy Agency c) Both 1 and 2
(IAEA): d) Neither 1 nor 2
1. The International Atomic Energy Agency is
entrusted with the task of upholding the principles
of the Non Proliferation Treaty of 1975. Q99. Consider the following statements regarding
‘Wheat’ :
2. India has recently become the 35th country to join
the IAEA Response and Assistance Network. 1. Wheat is rabi Crop and grown at a temperature of
10-26 °C with bright sunlight.
Which of the statements given above is/are
correct? 2. Russia and Ukraine accounted for nearly one-third
of global wheat exports.
a) 1 only
3. India is the second-largest producer of wheat but
b) 2 only accounts for less than 1% of the world’s wheat
c) Both 1 and 2 export.
d) Neither 1 nor 2 Which of the statements given above is/are
Q97. following statements regarding IT Rules, 2021 correct?
1. IT Rules, 2021 mandates a grievance redressal a) 1 and 2 only
system for over-the-top (OTT) and digital portals b) 2 only
in the country. c) 1 and 3 only
2. Social media platforms will also have to name a d) 1, 2 and 3
grievance officer who shall register the grievance
within 48 hours and dispose of it in 10 days.
3. Social media will have to publish an annual report
about the number of complaints received and the
status of redressal.
Which of the statements given above is/are
correct?
19

Q100. In which one of the following groups are all the


four countries included in the Organization of the
Petroleum Exporting plus(OPEC)?
a) Azerbaijan, Bahrain, Brunei, Ukraine
b) Malaysia, Mexico, Oman, Indonesia
c) Mexico, Oman, Russia, South Sudan
d) Bahrain, Brunei, Kazakhstan, Uzbekistan



PW Web/App: https://smart.link/7wwosivoicgd4
Download more eBooks & Test Series in Hindi and English
from our
Join Telegram- Telegram Channel-
https://t.me/upsc_success_time1
https://t.me/upsc_success_time1

Join Telegram Channel - Click Here


1

Indian Geography-Answers DPP 02

Q.1) Ans: D ❖ Statement 2 is incorrect: The standard meridian of


India passes through the following states:
Exp:
1. Uttar Pradesh
❖ Statement 1 is incorrect: Indira col is the
northernmost extreme point of India. It is situated 2. Madhya Pradesh
near Siachen Glacier in Ladakh. Whereas Indira
3. Chattisgarh
point is in the Great Nicobar at the southern point of
India. Kanyakumari is the southernmost extreme of 4. Orissa
India in the Indian mainland.
5. Andhra Pradesh
❖ Statement 2 is incorrect: Kibithu in Arunachal
Pradesh is the eastern most extreme point of India.
Q.4) Ans: D
❖ Statement 3 is correct: Guhar Moti in Gujarat is
the western most extreme point of India. Exp:

❖ Statement 1 is correct: Mizoram is also known as the


Molassis basin made of soft unconsolidated deposits.
Q.2) Ans: B
❖ Statement 2 is incorrect: Mahabharat range is
Exp:
located in Nepal above the Shivalik range. Hence, it
❖ Statement B is correct: Saramati Peak is situated in does not form a boundary between India and Nepal.
the state of Nagaland whereas Nokrek is situated in
❖ Statement 3 is incorrect: Geographically, Aksai
the state of Meghalaya. Annapurna and Makalu are a
Chin is an extension of the Tibet Plateau to the
part of Nepal, where Makalu is to the east of
southwest. It bordered by the Karakoram Range to the
Annapurna. Kamet peak and Nanda Devi lie in the
west and southwest and the Kunlun Mountains to the
state of Uttarakhand with Nanda devi to the east of
north and northeast. Thus it is not part of Karokaram
Kamet. Hence, the correct order from the east to
Range.
west direction is Saramati, Nokrek, Makalu,
Annapurna, Nanda Devi, and Kamet. ❖ Statement 4 is correct: Nubra valley is a triangular
valley formed with the confluence of river Nubra and
Shyok. It can be accessed through the Khardung La
Q.3) Ans: D pass, which is located in Leh region of Ladakh.

Exp:

❖ statement 1 is incorrect: The Indian Standard Q.5) Ans: B


Meridian (ISD) passes through the states of Uttar
Exp:
Pradesh, Madhya Pradesh, Chhattisgarh, Odisha,
and Andhra Pradesh. It passes an area in the eastern ❖ Statement 1 is incorrect: Bhabar is a narrow area
Andhra Pradesh. running parallel to the foothills of the Shivalik. It is
located at the end of its slope and hence river
deposits are associated with this area. The streams or
2

rivers usually disappear in this area. Terai areas are Garhjat range are the mountain ranges stretching
found south of the Bhabar areas and the streams or across Odisha, Jharkhand, and Chhattisgarh.
rivers emerge in these areas. Hence these areas are
more suitable for the cultivation of crops as
compared to Bhabar. Q.8) Ans: B

❖ Statement 2 is correct: The alluvial plains are Exp:


subdivided into Khadar and Bhangar. Bhangar are
❖ Statement 1 is incorrect: The presence of a wood
the older alluvium and are deposited away from the
fossil park at Atkal and the marine deposits of
flood plains of the rivers. Whereas Khadar are the
Brahmsar, Jaisalmer indicates the region was
new alluvium deposits which are subjected to
submerged under the sea and contains marine
frequent flooding. Hence, they are more fertile with
deposits. The underlying rocks of the region appear
finer silt deposition.
as an extension of the peninsular plateau.

❖ Statement 2 is correct: The Northern part of the


Q.6) Ans: C desert area is sloping towards Sindh while the
southern part has a slope towards Rann of Kachchh
Exp:
in Gujarat. Hence, the area shows opposing slopes.
❖ Statement 1 is incorrect: Vindhya and Kaimur
range are the source of origin of several rivers. Most
of the tributaries of Yamuna originate from here. Q.9) Ans: C
However, Banas River originates from the Aravallis
Exp:
and is an exception here.
❖ Option A is incorrect: Some parts of Lakshadweep
❖ Statement 2 is incorrect: Malda fault exists
Island lie above the 12-degree channel.
between the Rajmahal Hills/Chotanagpur Plateau
and the Meghalaya/Shillong Plateau. It is ❖ Option B is incorrect: Duncan passage in the
responsible for the detachment of the Northeastern Andaman is situated between south Andaman and
Plateau from the Deccan Plateau. The northeastern the Little Andaman.
plateau consists of Garo, Khasi, Jaintia hills in ❖ Option C is correct: Agatti Islands of Lakshadweep
Meghalaya and the Karbi Anglong hills of Assam. is situated to the north of the capital Kavaratti.

❖ Option D is incorrect: Saddle peak is the highest


Q.7) Ans: C peak of Andaman and is situated in the North
Andaman.
Exp:

❖ Statement C is correct: Harishchandra range is


located in the state of Maharashtra, Nallamala hills Q.10) Ans: C
form a part of the Eastern Ghats in Andhra Pradesh, Exp:
Dandakaranya is associated with Odisha,
Chhattisgarh, Telangana and Andhra Pradesh, ❖ Statement 1 is correct: The largest island in
Ramgarh hills are found in the state of Jharkhand, Lakshadweep is the Adrott Island. Minicoy island is
Malyagiri hills are located in the state of Odisha, and the second largest island of Lakshadweep.
3

❖ Statement 2 is correct: Minicoy Island is the range. It is a tributary of Ganga and meets Ganga at
southernmost Island of the Lakshadweep and is Prayagraj, Uttar Pradesh.
separated from the rest of the Lakshadweep through
❖ Statement 3 is incorrect: The river Son is the
the 9-degree channel.
largest south bank tributary of Ganga. It originates at
❖ the Amarkantak Plateau and joins Ganga at Arrah,
near Patna, Bihar.

❖ Statement 4 is incorrect: The river Kosi originates


Q.11) Ans: B
in Tibet, north of Mount Everest. It is joined by
Exp: rivers Son Kosi and Taimur Kosi and finally
emerges as Sapt Kosi after joining river Arun.
❖ Statement 1 is correct: The downward depression
created due to the Malda gap between the Rajmahal
hills and the Meghalaya plateau caused the Ganga
Q.13) Ans: A
and Brahmaputra rivers to drain to the Bay of
Bengal. Hence, it played a role in the evolution of Exp:
the Himalayan rivers.
❖ Statement 1 is correct: Kaziranga National Park, a
❖ Statement 2 is correct: Potwar Plateau or the Delhi World Heritage Site, is located in Assam. It is
ridge acted as a water divide between the Indus and known for one horned rhinoceros. River
the Ganga River system. Brahmaputra flows through this national park.

❖ Statement 3 is correct: The upliftment of western ❖ Statement 2 is correct: Pobitora Wildlife sanctuary
Himalayas also played a role in the creation of the in Assam is located on the south bank of river
present-day Himalayan drainage system which Brahmaputra. It houses the largest population
comprises the Indus, Ganga, and the Brahmaputra density of One-horned rhinoceros and is referred to
River system. as the Mini Kaziranga.

❖ Statement 4 is incorrect: Western coasts are an ❖ Statement 3 is incorrect: Nameri National Park is
example of submergent coast and it is for this reason located in Assam. It shares its boundary with the
that it has ideal conditions for the development of Pakhui Wildlife Sanctuary of Arunachal Pradesh. It
natural ports and harbors. It has no role in the has been declared as a Tiger Reserve. It does not lie
shaping of the present-day Himalayan River system. to the close vicinity of Brahmaputra.

Q.12) Ans: C Q.14) Ans: B

Exp: Exp:

❖ Statement 1 is correct: River Alaknanda finds its ❖ Statement B is correct: Among the given rivers,
source in the Satopanth Glaciers located above Rind, Sengar, and Sindh are the tributaries of
Badrinath. Yamuna. Raidak river is a tributary of Brahmaputra.
Son river is the second largest tributary of Ganga
❖ Statement 2 is correct: River Yamuna originates
after river Yamuna.
from the Yamunotri glacier near Bandarpunch
4

Q.15) Ans: B joining from left are the Vaghur, the Amravati, the
Buray, the Panjhra, the Bori, the Girna, the Purna,
Exp:
the Mona and the Sipna.
❖ Statement 1 is incorrect: Narmada and Tapi are the
❖ Statement 3 is correct: River Tapi flows through
west flowing rivers that flow through the rift valley
the rift valley. It also flows from west to east. (River
between the Vindhya and the Satpura ranges.
Narmada and Tapi both flow through the rift valley).
❖ Statement 2 is correct: Lacustrine (related to lakes)
❖ Statement 4 is incorrect: For the first 282 Km, the
are found from the Shivalik. They are important in
river flows in Madhya Pradesh, out of which 54 Km
tracing the evolution of the Himalayan River
forms the common boundary with Maharashtra
system.
State. It flows for 228 Km in Maharashtra draining
❖ Statement 3 is incorrect: There exists a slight (not Khandesh and east Vidarbha regions in the
sharp) tilt in the Deccan Plateau from west to east, northwest corner of the Deccan Plateau before
which is responsible for most of the rivers draining entering Gujarat.
the Bay of Bengal.

❖ Statement 4 is incorrect: River Tapi originates in


Betul, Madhya Pradesh and flows through the states Extra Edge by Only IAS
of Maharashtra, Madhya Pradesh, and Gujarat.
Rift valley
Nearly 79 percent of its drainage basin is in the state
of Maharashtra. A rift valley is a lowland region that forms where
Earth's tectonic plates move apart, or rift. Rift
valleys are found both on land and at the
Q.16) Ans: C bottom of the ocean, where they are created by
the process of seafloor spreading.
Exp:
Narmada and the Tapi are the major rivers in
❖ Statement 1 is incorrect: River Tapi originates near
India that flow through rift valleys.
Multai reserve forest in Betul district of Madhya
Pradesh. ❖ The Narmada originates on the western flank
of the Amarkantak plateau.

➢ Flowing in a rift valley between the


Satpura in the south and the Vindhyan
range in the north, it forms a picturesque
gorge in marble rocks and the Dhuandhar
waterfall near Jabalpur.

➢ Its catchment area is about 98,796 sq.


km.

➢ The Sardar Sarovar Project has been


❖ Statement 2 is correct: The important tributaries of constructed on this river.
Tapi river are the Suki, the Gomai, the Arunavati
❖ The Tapi is a westward flowing river.
and the Aner. Which joins it from right, and those
5

The united stream flows through the Pangi Valley


➢ It originates from Multai in the Betul parallel to the Pir Panjal Range.
district of Madhya Pradesh.
It meets the Indus at Panchnad in Pakistan.
➢ Nearly 79 per cent of its basin lies in
Maharashtra, 15 per cent in Madhya ❖ It is a left bank tributary of the Indus River.
Pradesh and the remaining 6 per cent in
Extra Edge by Only IAS
Gujarat.
Tributaries of Indus River

❖ Left bank – Zanskar River, Suru River, Soan


Q.17) Ans: B River, Jhelum River, Chenab River, Ravi
Exp: River, Beas River, Sutlej River, Panjnad
River, Ghaggar-Hakra River, Luni River.
❖ Option B is correct: River Indus: The Indus is a
transboundary river of Asia and a trans-Himalayan ❖ Right bank—. Shyok River, Hunza River,
River of South and Central Asia. The river rises in Gilgit River, Swat River, Kunar River, Kabul
Western Tibet, flows northwest through the Ladakh River, Kurram River, Gomal River, Zhob
and Gilgit-Baltistan regions of Kashmir, bends River.
sharply to the left after the Nanga Parbat massif, and
flows south-by-southwest through Pakistan, before it
empties into the Arabian Sea near the port city of
Karachi.

❖ Shyok :

❖ Right bank tributary of the Indus.

originates in Depsang Plains and flows along the


Karakoram Range.

Forms the eastern limit of the KarakoramFound


completely inside Jammu and Kashmir.

❖ Zaskar:

❖ Left bank tributary of the Indus.

flows between the Zaskar Range (S) and the Ladakh Q.18) Ans: C
Range (N).
Exp:
It meets the Indus just west of Leh.
❖ Option C is correct: River Ganga: Ganga River
Found completely inside Jammu and Kashmir. basin has the largest basin area in India. It is a
❖ Chenab: Himalayan River with a comprehensive network of
tributaries and distributaries. It is formed by two
It originates near the BaraLacha Pass as two headstreams head streams, Alaknanda and Bhagirathi which join
Chandra and Bhanga on the both sides of the pass. at Devprayag. Its left bank tributaries from west to
6

east are: Ramganga – Gomti– Ghaghara– Gandak – Exp:


Burhi Gandak- Kosi
❖ Statement 1 is correct: Peninsular rivers have been
➢ Its major right bank tributaries are Yamuna and Son. flowing for a long period. They have senile
topography and show features of mature stage. They
Extra Edge by Only IAS have almost reached the base level of erosion.
Gandak: ❖ Senile topography comprises old featureless
❖ Important tributary of Ganga. landforms – Rolling plateaus of Peninsular region
through which South Indian rivers flow.
❖ It originates near the Tibet-Nepal border.
❖ Statement 2 is incorrect: Rivers in peninsular
❖ Tributaries: Kali-Gandaki, Mayagadi, Bari, regions were not present before the Consequent
Trishula . landforms. The present landform has played an
❖ it Joins the Ganga at Hajipur, just east of important role in shaping the current drainage
Patna. pattern. This is called consequent drainage. Whereas
in Himalayan River the drainage pattern is
Gomati: antecedent which means it was present even before
❖ Important tributary of the Ganga, rises the creation of contemporary landforms.
from the Pilibhit district of UP and joins ❖ Statement 3 is correct: Unlike Himalayan rivers
the Ganga downstream of Sarnath. which flow through deep V-shaped valleys (gorges),
❖ It irrigates the Avadh Plain. Peninsular rivers flow in comparatively shallow
valleys, which are more or less completely graded.
Ramganga:
❖ Extra Edge by Only IAS
❖ Important tributary of the Ganga between
the Gomati (East) and the Ganga (West). ❖ The difference between the Himalayan and
peninsular rivers are tabulated below
❖ Rises in Garhwal district of Uttarakhand
and joins the Ganga near Kannauj. The Himalayan The Peninsular rivers
rivers
Ghaghra:
These rivers are These rivers are
❖ It originates near Gurla Mandhata peak,
perennial, that is they seasonal.
south of Mansarovar in Tibet. have water
throughout the year.
❖ It is Known as Karnali in western Nepal .
These rivers obtain The flow of these rivers
Tributaries:
water from rain as is based on rainfall.
❖ Sarda, Sarju, Rapti. well as from melted
snow from high
❖ It meets Ganga a few kilometers mountains.
downstream of Chapra in Bihar.
These rivers have These rivers have a
large river basins. smaller basin as
compared with the
Q.19) Ans: A Himalayan rivers.
7

Q.21) Ans: A
The rivers are mainly These rivers are
running in their considered as old rivers. Exp:
youth stage.
❖ Option A is correct: About the Watershed: An
The main source is The main source of the area drained by a river and its tributaries is called a
from glaciers of the river is the peninsular
drainage basin. The boundary lines separating one
Himalayan Mountain plateau and the central
range. highland. drainage basin from the other is known as the
watershed. The catchments of large rivers are called
river basins while those of small rivulets and rills are
Q.20) Ans: B often referred to as watersheds. There is a slight
difference between a river basin and a watershed.
Exp:
❖ Watershed is a small area while the basins cover
❖ Statement 1 is correct: Presence of Hard rocks is a
larger areas. River basins and watersheds are
reason behind non-formation of deltas as the west
marked by unity. What happens in one part of the
flowing rivers, especially the Narmada and the Tapi,
basin or watershed directly affects the other parts
flow through hard rocks and hence do not carry any
and the unit as a whole. Thus, they are accepted as
good amount of silt.
the most appropriate micro, meso or macro planning
❖ Statement 2 is correct: As west flowing rivers regions.
rapidly flow from steep slope and merges into
Arabian sea so they form estuaries and not deltas
Q.22) Ans: C
❖ Statement 3 is correct: The West flowing rivers
flow through faults (linear rift, rift valley, trough) so Exp:
much of the silt gets deposited there itself. Thus, less ❖ Option C is correct: River Chenab: The Chenab is
alluvium gives no opportunity in the formation of the largest tributary of the Indus. It is formed by two
delta. streams, the Chandra and the Bhaga, which join at
Tandi near Keylong in Himachal Pradesh. Hence, it
Extra Edge by Only IAS
is also known as Chandrabhaga.
Peninsular river
❖ The river flows for 1,180 km before entering into
❖ Rivers that drain into Bay of Bengal: The Pakistan. Important projects on Chenab River are
Mahanadi, the Godavari, the Krishna, the baglihar, dulhasti, and salar whereas Tulbul is
Cauvery and several smaller rivers drain important project on Jhelum River.
south-east into the Bay of Bengal.

❖ Rivers that drain into Arabian Sea: The


Narmada, the Tapi, the Mahi flowing west as
well as several small streams originating from
the Western Ghats flow westwards into the
Arabian Sea.
8

Q.23) Ans: D ❖ Ken River passes through Panna tiger reserve.

Exp:

❖ Option D is incorrectly matched: About the


Ganga: The Ganga is the most important river of
India both from the point of view of its basin and
cultural significance. It rises in the Gangotri glacier
near Gaumukh (3,900 m) in the Uttarkashi district of
Uttarakhand. Here, it is known as the Bhagirathi. It
cuts through the Central and the Lesser Himalayas in
narrow gorges.

❖ At Devprayag, the Bhagirathi meets the


Alaknanda; hereafter, it is known as the Ganga.
The Alaknanda has its source in the Satopanth
glacier above Badrinath. The Alaknanda consists of
the Dhauli and the Vishnu Ganga which meet at Q.25) Ans: D
Joshimath or Vishnu Prayag.
Exp:
❖ The other tributaries of Alaknanda such as the
❖ Statement 1 is correct: The Dnieper is one of the
Pindar joins it at Karna Prayag while Mandakini or
major rivers of Europe, rising in the Valdai Hills
Kali Ganga meets it at RudraPrayag.
near Smolensk, Russia, before flowing through
Belarus and Ukraine to the Black Sea. It is the
longest river of Ukraine and Belarus and the fourth-
Q.24) Ans: C
longest river in Europe, after the Volga, Danube,
Exp: and Ural rivers.
❖ Option C is correct: Ken-Betwa River interlinking
project: Ken-Betwa River interlinking project is the
first project under the National Perspective Plan for
interlinking of rivers. It envisages transferring water
from the Ken River to the Betwa River, both
tributaries of the Yamuna.

❖ The Ken-Betwa Link Canal will be 221 km long,


including a 2-km long tunnel.

❖ Ken and Betwa rivers originate in MP and are the


tributaries of Yamuna.

❖ Ken meets with Yamuna in Banda district of UP and ❖ Statement 2 is incorrect: The Volga is the longest
with Betwa in Hamirpur district of UP. river in Europe. Situated in Russia, it flows through
Central Russia to Southern Russia and into the
❖ Rajghat, Paricha and Matatila dams are over Betwa Caspian Sea. It is also Europe's largest river in terms
river.
9

of average discharge at delta and of drainage basin.


It is widely regarded as the national river of Russia. Extra Edge by Only IAS

Consequent Drainage Rivers:

❖ A consequent drainage system is when the river


follows the general direction of the slope.

❖ Rivers of Peninsular India such as the Godavari,


Krishna, and Cauvery are consequent rivers
descending from the western ghats and flowing
into the Bay of Bengal.

Superimposed drainage:
❖ Statement 3 is correct: The Danube is the second- ❖ A river flowing over a softer rock stratum
longest river in Europe, after the Volga in Russia. It reaches the harder basal rocks but continues to
flows through much of Central and Southeastern follow the initial slope.
Europe, from the Black Forest into the Black Sea.
❖ The stream has enough erosive power that it cuts
its way through any kind of bedrock,
maintaining its former drainage pattern.

❖ It exhibits discordance with the underlying rock


structure as it originally developed on a cover of
rocks that have now disappeared and retains
their courses unaffected by the newly exposed
structures.

❖ Examples of Superimposed drainage or rivers –


The Damodar, the Subarnarekha, the Chambal,
the Banas, and the rivers flowing at the Rewa
Q.26) Ans: A
Plateau.
Exp:
Subsequent River System:
❖ Option A is correct: Antecedent Drainage: An
❖ A subsequent river is streams that have
antecedent stream is a stream that maintains its
generally developed after the original stream
original course and pattern despite the changes in
(Consequent River).
underlying rock topography.
❖ It is a tributary stream that is eroded along an
❖ Many Himalayan rivers are good examples of
underlying belt of non-resistant rock after the
antecedent origin. These rivers originated well
main drainage pattern has been established.
before the Himalayan region was uplifted. The rivers
Indus, Brahmaputra, Sutlej, Kosi and Subansiri ❖ For example – Chambal, Sind, Ken, Betwa,
originated on the Tibetan side and now traverse the Tons, and Son are the subsequent drainage
existing mountain ranges, cutting deep gorges. rivers and meet the Yamuna and the Ganga at
right angles (the main drainage).
10

Q.27) Ans: C ❖ Statement 4 is incorrect:

Exp: ❖ The prime reason for the dry conditions of the Thar
Desert is that while the Bay of Bengal branch of the
❖ Statement 1 is incorrect: The North Eastern
monsoon sheds most of its moisture by the time it
Monsoon has little to zero correlation with the
reaches the Thar Desert, the Arabian Sea branch of
Position of the Himalayas. They are in fact a
the monsoon winds blow almost parallel to the the
manifestation of the global wind belts.
Aravali ranges and hence more or less bypass
❖ The easterlies blow naturally between 0 degrees maximum parts of Rajasthan.
(equator) and 30 degrees Latitude. Hence even if
❖ Himalayas therefore have little to zero role to play
there were no Himalayas, the north eastern easterlies
here.
would still have picked up moisture from the Bay of
Bengal. The shores of Tamil Nadu would still get
the same amount of rainfall from the North Eastern
Monsoon in the month of December and January.

❖ Statement 2 is incorrect: The onset of Monsoon in


India takes place due to the seasonal reversal of
winds. The South Western Monsoon winds originate
from MASCARENE (near Madagascar in Western
Indian Ocean) and are pulled by the low-pressure
prevailing (due to excess heating) over the Tibetan
Highlands during the summer season.

❖ Thus, even if The Himalayas were absent, the


seasonal reversal would still have taken place and Q.28) Ans:
the onset of the monsoon would occur at its usual Exp:
time (around June 1st week).
❖ All options are correct:
❖ Statement 3 is correct: One of the most important
ways in which the Himalayas influence the Indian ❖ Sea surface temperature (SST) is routinely used for
climate Is by trapping the monsoon winds, forcing predicting whether the total amount of rainfall that
them to shed their moisture within the subcontinent. India receives during the monsoon season will be
less or more than the long-term mean of 887.5 mm.
❖ Had there been no Himalayas, the South Western The information on whether the amount of monsoon
monsoon winds would have had an almost rainfall will be more or less than the long-term mean
uninterrupted passage over the Indian mainland. is generally made available by the beginning of
This would leave little scope for any kind of April. The Earth System Science Organisation
orographic rainfall over the Indian subcontinent. (ESSO) and the India Meteorological Department
❖ Thus, the Long Period Average value of the rainfall (IMD) take into account five conditions while
recorded in India would be lower than the present making forecasts about the monsoon in April every
value (89cm) as other mountain ranges (western year.
Ghats, Satpuras and the Vindhyas) would still cause
some, if not much, orographic rainfall.
11

❖ The SST gradient between the north Atlantic and the days.
north Pacific (conditions during December of the
previous year and January of present year)

❖ SST of the equatorial south Indian Ocean


(conditions during February and March of the
present year)

❖ Mean Sea Level pressure in east Asia (conditions


during February and March of the present year)

❖ The surface air temperature over northwest Europe


(conditions during January of the present year)

❖ The warm water volume of the equatorial Pacific


❖ Indian Summer Monsoon Rainfall exhibits a
Ocean (conditions during February and March of the
prominent inter-annual variability. A season of
present year)
deficient June to September monsoon rainfall in
❖ In recent times however, scientists are increasingly India is followed by warm sea surface temperature
of the opinion that ocean mean temperature (OMT) (SST) anomalies over the tropical Indian Ocean and
has a better ability to predict this than sea surface which generally yields normal or excessive rainfall
temperature. Reasons : the following season.

❖ Sea surface temperature gives information only ❖ The intraseasonal variability of rainfall during a
about the thin upper layer of the ocean and does not monsoon season is characterized by the occurrence
reflect the thermal energy available in the upper of active and break phases.
ocean

❖ The SST is restricted to a few millimetres of the top


ocean layer and is largely influenced by strong
winds, evaporation, or thick clouds

❖ The variations in the upper ocean thermal energy


conditions are mainly responsible for the summer
monsoon.

Q.29) Ans: A

Exp:
❖ During the active phase, the rainfall is above normal
❖ Options 1, 2 and 4 are correct:
over central India and below normal over northern
❖ The Monsoonal winds typically burst onto the India (foothills of the Himalaya) and southern India.
shores of Kerala in the 1st week of JUNE. This This pattern is reversed during the break phase (also
means that at the time of its arrival, the normal called Monsoon Breaks).
rainfall increases suddenly and continues constantly
for several
12

❖ Option 3 is incorrect: Western Disturbances are are characterized by subsidence of wind) system
phenomena independent of the phenomenon/ towards the east of the Mediterranean Sea across
mechanism of Monsoon season in India. A western Afghanistan, Pakistan and North West India. These
disturbance is an extratropical storm originating in storms are residual frontal cyclones which move at
the Mediterranean region that brings sudden winter the height of 2000 metres from the mean sea level.
rain to the northern parts of the Indian subcontinent.
❖ Statement 3 is correct: Rabi crops during the
It is a non-monsoonal precipitation pattern driven by
winter season are greatly influenced by the arrival of
the westerlies.
western disturbance. It brings cloudiness and
❖ Option 5 is not correct: The withdrawal of precipitation in many parts of north India during rabi
monsoon in India is a gradual process, Unlike the season. Rabi crops, particularly wheat crop, down
onset of the monsoonal winds. The North Eastern under rainfed conditions.
trade winds gradually push the southwestern
westerlies as the Intertropical convergence zone
(ITCZ) starts moving southwards in September. Q.31) Ans: A

Exp:

Q.30) Ans: C ❖ Statement 1 is correct and 2 is incorrect: An MJO


is an oceanic-atmospheric phenomenon which
Exp:
affects weather activities across the globe. It brings
❖ Statement 1 is incorrect: Western disturbances are major fluctuation in tropical weather on weekly to
weak temperate or Extra-Tropical (not tropical) monthly timescales.
frontal cyclones that originate over the
❖ The MJO can be defined as an eastward moving
Mediterranean Sea, gain moisture from the Caspian
'pulse' of clouds, rainfall, winds and pressure near
Sea and the Black Sea and cause winter rainfall in
the equator that typically recurs every 30 to 60 days.
the North Western part of India. The Himalayas act
It is a traversing phenomenon and is most prominent
as orographic barriers.
over the Indian and Pacific Oceans.

❖ Statement 3 is incorrect: The Indian Ocean Dipole


❖ Statement 2 is incorrect: The subtropical westerly
(IOD), El Nino and MJO are all oceanic and
jet stream drives this low pressure (cyclones cannot
atmospheric phenomena, which affect weather on a
develop in High pressure regions as these regions
large scale. The El-Nino is known to cause droughts
13

in India. However, if the periods of El Nino overlap 2020, north India did not experience such
with the convective phase of the Madden Julian temperatures till May 21.
oscillations, the effect of El Nino can be reduced to
❖ It was mainly because of the continuous inflow of
some extent especially in the Arabian Sea coast
Western Disturbances that influenced the weather in
regions where it rains heavily when the MJO passes
the north till as late as April. There has been
over the Indian Ocean. But the El Nino’s impact is
frequent passing of Western Disturbances over the
felt across India while the madden-julian
north, appearing after every five to seven days.
oscillation’s impact is limited to only some parts of
Originating in the Mediterranean Sea, Western
India for a period of 30 to 60 days. Therefore, a
Disturbances are eastward-moving winds that blow
complete nullification is not possible.
in lower atmospheric levels. They thus affect the
local weather of a region during its onward journey.

Q.32) Ans: B ❖ Option D is correct: In 2020, Cyclone Amphan,


which hit the coasts of Northern Odisha and West
Exp:
Bengal, had a huge role in triggering heat waves
❖ Option A 1 is correct: Heat Waves occur over India across North India. Cyclone Amphan, which was a
between March and June. Meteorologists define a massive Super Storm covering 700 kms, managed to
heatwave event in reference to the normal maximum drag maximum moisture from over the Bay of
(day) temperature for a location. Bengal, entire South Peninsula, parts of Central
India and to some extent, even from the Arabian Sea
❖ For example, Heatwaves are declared in the plains if
the temperature crosses 40 degrees Celsius. Over the ❖ All the moisture that was otherwise built during the
hills, the threshold temperature is 30 degrees thunderstorm and rainfall, got gradually depleted
Celsius. When the day temperature jumps by 4 to 5 from over vast areas as the storm advanced towards
degrees above the normal maximum temperature of West Bengal and Bangladesh. It thus triggered dry
a location, it is declared as a heatwave. north-westerly winds to blow over Rajasthan,
Punjab, Madhya Pradesh, Uttar Pradesh and
❖ Option B is incorrect: Heatwaves are common over
Maharashtra causing severe heatwave.
the Core Heatwave Zone (CHZ) — Rajasthan,
Punjab, Haryana, Chandigarh, Delhi, West Madhya
Pradesh, Uttar Pradesh, Chhattisgarh, Orissa,
Q.33) Ans: C
Vidarbha in Maharashtra, parts of Gangetic West
Bengal, Coastal Andhra Pradesh and Telangana, as Exp:
categorised by India Meteorological Department.
❖ Option C is correct:, The Monsoon hits the
❖ The regions in the extreme north, northeast and Andaman and Nicobar Islands, a full week before
southwestern India are less prone to heatwaves. Heat the Arabian Sea branch hits the Kerala coast
waves are rare over the peninsular region because of (mainland of India).
Maritime conditions.
❖ Western Rajasthan is the last place (Mid July) where
❖ Option C is correct: Summer season reaches its the monsoon reaches - which partially explains the
peak by May 15 in India, when the day temperatures arid conditions.
across north, west, and central India cross 40
degrees and hover close to 45 degrees then on. In
14

Q.34) Ans: D rain over Northwest India and sometimes, other


parts of North India.
Exp:
❖ Statement 2 is correct: Western Disturbances are
❖ Pair 1 is correct: Mango Shower: Towards the end
the cause of the most winter and pre-monsoon
of summer, there are pre-monsoon showers which
season rainfall across North-West India. This
are a common phenomenon in Kerala and coastal
phenomenon is usually associated with cloudy sky
areas of Karnataka. Locally, they are known as
and unusual rain. Moreover, an increase in the
mango showers since they help in the early ripening
prevailing night temperature generally indicates an
of mangoes.
advance in the arrival of these cyclone disturbances.
❖ Pair 2 is incorrect: Nor Westers: These are
dreaded evening thunderstorms in Bengal and
Assam. Their notorious nature can be understood Q.36) Ans: A
from the local nomenclature of ‘Kalbaisakhi’, a
Exp:
calamity of the month of Baisakh. These showers
are useful for tea, jute and rice cultivation. In ❖ Pair 1 is correct: According to Koeppen, Tropical
Assam, these storms are known as “Bardoli climates are climatic types where the mean monthly
Chheerha”. temperature throughout the year is over 18°C.

❖ Pair 3 is correct: Loo: Hot, dry and oppressing ❖ Pair 2 is incorrect: Warm temperate climates are
winds blowing in the Northern plains from Punjab to regions, where the mean temperature of the coldest
Bihar with higher intensity between Delhi and Patna. month is between 18°C and minus 3°C.

❖ Pair 3 is incorrect: Ice climates for particular


regions are where the mean temperature of the
Q.35) Ans: B
warmest month is under 10°C.
Exp:

❖ Statement 1 is incorrect: Western Disturbances


develop in the mid latitude region (north of the Extra Edge by Only IAS
Tropic of Cancer), not in the tropical region, Koeppen based his scheme of Climatic
therefore they are called as mid latitude storms or classification on monthly values of
extra-tropical storms. The western cyclonic temperature and precipitation. He
disturbances are weather phenomena of the winter identified five major climatic types,
months brought in by the westerly flow from the namely:
Mediterranean region. They usually influence the
weather of the north and north-western regions. The 1. Tropical climates, where the mean monthly
low pressure typically forms over the Mediterranean temperature throughout the year is over
Sea and travels over Iran, Iraq, Afghanistan and 18°C.
Pakistan before entering India loaded with moisture. 2. Dry climates, where precipitation is very
These moisture laden western disturbances low in comparison to temperature, and
eventually come up against the Himalayas and get hence, dry. If dryness is less, it is semiarid
blocked, as a consequence, the moisture gets trapped (S); if it is more, the climate is arid(W).
and precipitation is shared in the form of snow and
15

Exp:
3. Warm temperate climates, where the mean
temperature of the coldest month is between ❖ Option D is correct: Laterite Soil in India: The
18°C and minus 3°C. laterite soils develop in areas with high temperatures
and high rainfall. These are the result of intense
4. Cool temperate climates, where mean leaching due to tropical rains. With rain, lime and
temperature of the warmest month is over silica are leached away, and soils rich in iron oxide
10°C, and mean temperature of the coldest and aluminum compounds are left behind. Humus
month is under minus 3°C. content of the soil is removed fast by bacteria that
5. Ice climates, where the mean temperature of thrive well in high temperatures.
the warmest month is under 10°C. ❖ These soils are poor in organic matter, nitrogen,
phosphate, and calcium, while iron oxide and potash
are in excess. Hence, laterites are not suitable for
Q.37) Ans: B cultivation; however, the application of manures and
fertilizers are required for making the soils fertile for
Exp:
cultivation.
❖ Statement 1 is correct: Variability of rainfall in
❖ Red laterite soils in Tamil Nadu, Andhra Pradesh,
monsoon brings droughts or floods every year in
and Kerala are more suitable for tree crops like
some parts of the country. These regional variations
cashew nuts. Laterite soils are widely cut as bricks
in monsoon climate help in growing various types of
for use in house construction. These soils have
crops. Except Himalayas, all the parts of the country
mainly developed in the higher areas of the
have temperatures above the threshold level to grow
Peninsular plateau.
the crops or plants throughout the year.

❖ Statement 2 is correct: Sudden monsoon bursts


create a problem of soil erosion over large areas in Q.39) Ans: A
India. As, the summer rainfall causes considerable
Exp:
runoff and soil erosion. Moreover, due to global
warming, The temperature of the world is ❖ Statement 1 is incorrect: Red soil develops on
significantly increasing. Carbon dioxide produced crystalline igneous rocks in areas of low rainfall in
by human activities is a major source of concern that the southern and eastern part of the Deccan Plateau.
negatively affects the rain pattern in the Indian A long stretch of area is occupied by red loamy soil,
subcontinent. along the piedmont zone of Western Ghat. Yellow
and red soils are also found in parts of Chhattisgarh
❖ Statement 3 is correct: Agricultural prosperity of
and Odisha and in the southern parts of the middle
India depends very much on timely and adequately
Ganga plain.
distributed rainfall. If it fails, agriculture is adversely
affected particularly in those regions where means ❖ Statement 2 is correct: Due to a wide diffusion of
of irrigation are not developed. iron in crystalline and metamorphic rocks, the soil
develops a reddish color. When it occurs in a
hydrated form, it looks yellow. The fine-grained red
Q.38) Ans: D and yellow soils are normally fertile, whereas in dry
upland areas the soil has poor fertility due to coarse-
16

grained structure. They are generally poor in humus, cyclonic activity, typical of Britain, and are hence
phosphorus, and nitrogen. named British type of climate.

❖ Britain, lowlands North-West Europe, northern and


western France, Belgium, the Netherlands,
Q.40) Ans: B
Denmark, western Norway and also north-western
Exp: Iberia.

• Option B is correct: About the Black Soils: Black ❖ In the southern hemisphere, southern Chile,
soil covers most of the Deccan Plateau which Tasmania, and most parts of New Zealand. The
includes parts of Maharashtra, Madhya Pradesh, mean annual temperatures are usually between 5°C
Gujarat, Andhra Pradesh and some parts of Tamil and 15°C. The annual range of temperature is small.
Nadu. In the upper reaches of the Godavari and the Summers are, in fact, never very warm.
Krishna, and the north western part of the Deccan
❖ The British type of climate has adequate rainfall
Plateau, the black soil is very deep.
throughout the year with a tendency towards a slight
❖ These soils are also known as the ‘Regur Soil’ or the winter or autumn maximum from cyclonic sources
‘Black Cotton Soil’. The black soils are generally
❖ Since the rain-bearing winds come from the west,
clayey, deep and impermeable. They swell and
the western margins have the heaviest rainfall. The
become sticky when wet and shrink when dried. So,
amount decreases eastwards with increasing distance
during the dry season, these soils develop wide
from the sea.
cracks. Thus, there occurs a kind of ‘self-
ploughing’. ❖ Natural Vegetation:

❖ Because of this character of slow absorption and loss ❖ The natural vegetation of this climatic type is
of moisture, the black soil retains the moisture for a deciduous forest. The trees shed their leaves in the
very long time, which helps the crops, especially the cold season, to protect themselves against the winter
rain fed ones, to sustain even during the dry season. snow and frost. The common species include oak,
Chemically, the black soils are rich in lime, iron, elm, ash, birch, beech, poplar, and hornbeam.
magnesia and alumina. They also contain potash.
❖ Unlike the equatorial forests, the deciduous trees
But they lack phosphorus, nitrogen and organic
occur in pure stands and have greater lumbering
matter. The colour of the soil ranges from deep
value from the commercial point of view.
black to grey.
❖ The deciduous hardwoods are excellent for both fuel
and industrial purposes.
Q.41) Ans: A

Exp:

❖ Option A is correct: British Type Climate: The


Cool Temperate Western Margin (British Type)
Climate: The cool temperate western margins are
under the permanent influence of the Westerlies all-
round the year. They are also regions of much
17

and parasitic plants that live on other plants. Under


Extra edge by Only IAS the trees grow a wide variety of ferns, orchids and
Mediterranean Type: It is a warm temperate lalang.
western-margin climate. It rains here in the ❖ Statement 2 is incorrect: The equatorial regions are
winter, and the summers are dry and warm. generally sparsely populated. In the forests most
Steppe Type: It is a temperate continental type primitive people live as hunters and collectors and
of climate with little rainfall and the the more advanced ones practise shifting cultivation.
natural vegetation is grass (varying in density Food is so abundant in such a habitat that many
and quality). people worry very little about the life of the next
day.
Natal Type: This is experienced in the warm
temperate eastern margin (non-monsoonal areas)
of the southern hemisphere. Q.43) Ans: C

Exp:

Q.42) Ans: A ❖ Statement 1 is correct: The Himalayan ranges


show a succession of vegetation from the tropical to
Exp:
the tundra, which change in with the altitude.
❖ Equatorial Vegetation: High temperature and Deciduous forests are found in the foothills of the
abundant rainfall in the equatorial regions support a Himalayas. It is succeeded by the wet temperate
luxuriant type of vegetation-the tropical rainforest. type of forests between an altitude of 1,000-2,000 m.
In the Amazon lowlands, the forest is so dense and In the higher hill ranges of northeastern India, hilly
so complete in its- vegetation extravagance that a areas of West Bengal and Uttaranchal, evergreen
special term 'selvas' is used. broadleaf trees such as oak and chestnut are
predominant. Between 1,500-1,750 m, pine forests
❖ Unlike the temperate regions, the growing season
are also well-developed in this zone, with Chir Pine
here is all year round- seeding, flowering, fruiting
as a very useful commercial tree.
and decaying do not take place in a seasonal pattern,
so some trees may be in flower while others only a ❖ Statement 2 is correct: Deodar, a highly valued
few yards away may be bearing fruit. There is endemic species, grows mainly in the western part
neither drought nor cold to check growth in any part of the Himalayan range. Deodar is a durable wood
of the year. The characteristic _ features of the mainly used in construction activity. Similarly, the
equatorial vegetation may be summarized as chinar and the walnut, which sustain the famous
follows. Kashmir handicrafts, belong to this zone.

❖ Statement 1 is correct: 1. A great variety of


vegetation: The equatorial vegetation comprises a
Q.44) Ans: C
multitude of evergreen trees that yield tropical
hardwood, e.g., mahogany, ebony, greenheart, Exp:
cabinet woods and dyewoods. There are smaller
❖ Option C is correct: Tropical Dry deciduous forest
palm trees, climbing plants like the lianas or rattan
covers: Tropical Dry deciduous forest covers vast
which may be hundreds of feet long and epiphytic
areas of the country, where rainfall ranges between
18

70 -100 cm. On the wetter margins, it has a soon wash out most of the soil nutrients and the soil
transition to the moist deciduous, while on the drier deteriorates rapidly.
margins to thorn forests. These forests are found in
❖ The equatorial conditions are ideal for the survival
rainier areas of the Peninsula and the plains of Uttar
of germs and bacteria and are easily transmitted
Pradesh and Bihar.
through moist air. Insects and pests not only spread
❖ In the higher rainfall regions of the Peninsular diseases but are injurious to crops. Eg. Coffee Berry
plateau and the northern Indian plain, these forests Diseases.
have a parkland landscape with open stretches in
which teak and other trees interspersed with patches
of grass are common. Q.46) Ans: D

❖ As the dry season begins, the trees shed their leaves Exp:
completely and the forest appears like a vast
❖ Statement 1 is correct: India has a rich variety of
grassland with naked trees all around. Tendu, palas,
wetland habitats. About 70 percent of this comprises
amaltas, bel, khair, axlewood, etc. are the common
areas under paddy cultivation. The total area of
trees of these forests. In the western and southern
wetland is 3.9 million hectares. Two sites- Chilika
part of Rajasthan, vegetation cover is very scanty
Lake (Odisha) and Keoladeo NationalPark
due to low rainfall and overgrazing.
(Bharatpur) are protected as water-fowl habitats
under the Convention of Wetlands of International
Importance (Ramsar Convention).
Q.45) Ans: B
❖ Statement 2 is incorrect: In India, the mangrove
Exp:
forests spread over 6,740 sq. km which is 7 percent
❖ Option B is correct: Plantations in the Equatorial of the world’s mangrove forests. They are highly
Regions: With the process of colonization, many developed in the Andaman and Nicobar Islands and
large plantations have been established in the the Sundarbans of West Bengal. Other areas of
equatorial regions of the world. significance are the Mahanadi, the Godavari and the
Krishna deltas. These forests too, are being
❖ The hot and wet climate has proved to be very
encroached upon, and hence, need conservation.
favourable for the cultivation of certain crops like
rubber, cocoa, tea, etc. that are highly valued in the ❖ Statement 3 is correct: Mangroves grow along the
industrial world. coasts in the salt marshes, tidal creeks, mud flats and
estuaries. They consist of a number of salt-tolerant
❖ High temperature and abundant rainfall support a
species of plants. Crisscrossed by creeks of stagnant
luxuriant type of vegetation-the tropical rainforest.
water and tidal flows, these forests give shelter to a
These forests are spread throughout the equatorial
wide variety of birds
region. In fact, these forests are so dense and widely
spread, that some regions like the Amazon are
described as the 'lungs of the earth'.
Q.47) Ans: D
❖ Despite having heavy leaf-fall and the
Exp:
decomposition of leaves by bacteria, the soil here is
not very nutrient-rich, as the torrential downpours ❖ All Options are correct
19

(Hyderabad-Karnataka), Telangana,
Andhra Pradesh and Tamil Nadu.

Q.48) Ans: B

Exp:

❖ Ebony and Mahogany are examples of hardwood


trees found in Evergreen Forests.

➢ Thorny Scrub Forests of India

✓ These are forests which receive rainfall


less than 50 cm annually.

✓ These consist of open woodland with


thorny trees with short trunks and low, ❖ Options 1 and 3 are correct:
branching crowns; spiny and xerophytic
shrubs; and dry grassland . ❖ Dandeli anshi (karnataka, Western ghats) and
Cambell bay (Great nicobar) contain evergreen
✓ The trees are low and widely scattered. vegetation.
Acacias and Euphorbias are very
prominent along with Khairm babool, ❖ Options 2 and 4 are incorrect
neem and palas etc. ❖ Mukundara hills (Rajsthan) and Gugamal MP
✓ This is the habitat of the great Indian (Maharasthra) fall under the dry deciduous
bustard and blackbuck, vegetation category in india.

✓ The remaining natural habitat is threatened ❖ While Gugamal NP receives moderate rainfall due
by overgrazing and invasive weeds to its location in the leeward side of the Western
Ghats, the Mukundara hills NP receive medium
✓ TRajasthan, south-western Punjab, western rainfall due to its interior location in the Central
Haryana, Kutch and neighbouring parts of highlands of India.
Saurashtra (gujrat) .

✓ Such forests also grow on the leeside of


the Western Ghats covering large areas of Q.49) Ans: B
Maharashtra (Vidarbha), Karnataka Exp:
20

Shelterbelts: Exp:

❖ Statement 1 is incorrect: The moist deciduous


forests are found in the regions, which record
rainfall between 100 and 200 cm. The trees drop
their leaves during the spring and early summer
when sufficient moisture is not available (Autumn is
the season between summers and winters). This
process is known as Abscission. They are also called
‘Monsoon forests.’

❖ Statement 2 is correct: As can be seen in the table,


❖ In general, shelterbelts (windbreaks) are strips of Total area covered by moist deciduous forests is
vegetation composed of trees and shrubs grown almost 25 times the area covered by Littoral and
along the coasts to protect coastal areas from high Swamp Forests.
velocity winds and also from devastations like the
ones caused by tsunami.

❖ Example: Sunderbans act as natural shelterbelts in


West Bengal and help minimize the impact of
various cyclones in the area.

❖ They also enhance the biodiversity in the area while


helping enhance soil fertility and health too.
Agriculturists have a long history of enhancing crop
growth by manipulating soil and plant
❖ Statement 3 is correct: These forests exist mostly
microclimates, through use of irrigation,
in the eastern part of the country – northeastern
glasshouses, shelterbelts and windbreaks, snow
states, along the foothills of the Himalayas,
fences, wind machines, surface mulches, certain
Jharkhand, Andhra Pradesh, West Orissa and
tillage practices, alley cropping, and agroforestry.
Chhattisgarh, and on the eastern slopes of the
They also serve the purpose of sand binders and
Western Ghats.
prevent sand erosion.
❖ The Eastern slopes of the Southern Western Ghats
receive rainfall from both the South West monsoon
and Northeast Monsoon. Similarly, Parts of Coastal
Andhra Pradesh are also influenced by rains during
both the seasons.

❖ The interesting thing here is despite getting rains


during both the Southwest monsoon and the
Northeast monsoon, the amount of rain received is
not enough (for them to be categorized as Evergreen
Forests) and hence tree leaves are invariably shed
during some point in the spring season.
Q.50) Ans: C
21

❖ Both Arunachal Pradesh and Uttarakhand have large


swathes of Tropical Pine forests (evergreen) in the
Q.51) Ans: C
lower Himalayas and consists of Alpine vegetation
Exp: such as perennial grasses, sedges, cushion plants,
mosses, and lichens above the treeline in the upper
❖ Statement 1 is correct: Tropical Evergreen and
Himalayas.
Tropical Dry Deciduous forests coexist in states
such as Maharashtra and Karnataka. While the ❖ Statement 4 is incorrect
leeward side of the Western Ghats consists of
❖ As can be seen in this table given by the Forest
tropical dry deciduous forests and thorn and Scrub
Survey of India, Goa, Lakshadweep and Puducherry
forests, the windward side is primarily made up of
have zero or negligible scrub forest vegetation.
Wet Evergreen forests.

❖ Statement 2 is correct: States such as Kerala and Q.52) Ans: D


West Bengal consist of both wet evergreen forests
and mangrove forest Exp:

❖ In Bengal, Tropical evergreen forests occur in the ❖ Option D is correct: Assessment of biodiversity in
terai regions of Darjeeling and Jalpaiguri regions; forests is important since it provides an indicator to
While the Sunderbans are famous for their represent the state of conservation of forest
Mangroves. ecosystems and it can help to evaluate and monitor
sustainability of the biological resources. It also
❖ In Kerala, while areas under the Western ghats helps in comparative evaluation of stability,
constitute the Evergreen forests, the coastal districts productivity and ecosystem functions of forests in
of Kannur, Kozhikode, Ernakulam and Alleppey temporal and spatial scales.
have limited but important swathes of Mangrove
Forests. ❖ Several quantitative indices have been designed to
provide information on different aspects of
❖ Statement 3 is correct: States such as Uttarakhand biodiversity viz, Margalef index, Menhinick index,
and Arunachal Pradesh consist of both tropical Simpson index, Shannon-Wiener Index etc. The
evergreen forests and Alpine vegetation. most commonly used index is the Shannon-Wiener
22

Index which is based on information theory that


provides the biodiversity values and helps to
Q.54) Ans: A
compare it between plant communities/ecosystems.
It gives a measure of species abundance and richness Exp:
to quantify diversity of the species.
❖ Statements 1,2 and 4 are correct: Foliar
feeding/application refers to the spraying of fertilizer
solutions containing one or more nutrients on the
Q.53) Ans: B
foliage of growing plants. The liquid fertilizer is
Exp: directly applied to the leaves as opposed to in the
soil. The absorption takes place through their
❖ Statements 1,3 and 4 are correct:
stomata and their epidermis. In some cases, Plants
ALLELOPATHY and its Applications: Allelopathy
are also able to absorb nutrients through their bark.
is a biological phenomenon, “a chemical warfare,”
among the plants imposed by one plant on another to ➢ Advantages of Foliar Applications
suppress the latter and take advantage of that
✓ A foliar application has proven to be an
suppression. An organism produces one or more
excellent method of supplying plant
biochemicals that influence the germination, growth,
requirements for secondary nutrients
survival, and reproduction of other organisms.
(calcium, magnesium, and sulfur) and
Example: plants release chemical compounds from
micronutrients (zinc, manganese, iron,
their roots into the soil, and these chemicals
copper, boron, and molybdenum), while
suppress or even kill the neighboring plants when
supplementing N-P-K needs.
they are absorbed by the plants.
✓ Leaching is the loss of soluble substances
❖ Various Applications of Allelopathy are:
and colloids from the top layer of soil by
❖ Weed Management: Allelopathic extracts have percolating precipitation. Foliar
been exploited for weed control. application has been shown to avoid the
problem of leaching-out in soils and
❖ Disease Management: Aqueous extract of many
prompts a quick reaction in the plant.
allelopathic plants is known to exhibit antifungal
properties. Eg: leaf extracts of jimson weed (Datura ✓ Foliar sprays give better control over
stramonium) reduced the development of yellow rust concentration and frequency that optimally
pustules on the leaves of wheat. addresses and resolves any nutrient
deficiency. However, even in Direct
❖ Insect-Pest Control: allelopathy has also a potential
application into the root zone, the modern-
in managing insect pests. Neem (Azadirachta indica)
day techniques can more or less control the
and its extracts have been used for decades for
concentration of various fertilizers.
managing stored grain and other insect pests.
✓ The nutrients applied can be tank-mixed
❖ Resistance Against Abiotic Stresses: Focused
with pesticides, thereby achieving a
interdisciplinary long-term research has shown that
synergic effect and additional saving on
allelopathy can boost the yield of crop plants by
application costs.
minimizing the vagaries of biotic and abiotic
stresses.
23

✓ Foliar feeding can serve as a timely for highest economic benefits. In a nursery, weeds
measure to prepare the crops to expect do not affect the rice since the environment is
biotic or abiotic stress, such as chill, controlled.
drought, or heat stresses.
❖ The seed requirement for DSR is also high, 8-10
✓ The high efficacy offers considerable kg/acre, compared to 4-5 kg/acre in transplanting.
reduction of the application rate (as
❖ Further, laser land levelling is compulsory in DSR.
compared to soil application), without
This is not so in transplanting.
compromising the results, thus a marked
saving on the fertilizer is achieved and a ❖ The sowing needs to be done timely so that the
minimal environmental foot-print takes plants have come out properly before the monsoon
place. rains arrive.

Q.55) Ans: A Q.56) Ans: D

Exp: Exp:

❖ Statements 1 is correct: DSR refers to the process ❖ Option D is correct: Seaweeds: Weeds are
of establishing a rice crop from seeds sown in the unwanted and undesirable plants that interfere with
field rather than by transplanting seedlings from the utilization of land and water resources and thus
nursery. Direct seeding can be done by sowing pre- adversely affect crop production and human welfare.
germinated seed into a puddled soil (wet seeding) or They are gregarious in nature compared to crop
standing water (water seeding) or prepared seedbed plants.
(dry seeding).
❖ Uses of seaweeds are as follows:
❖ Direct-seeded plants mature 7 to 10 days earlier than
❖ Seaweeds are important as food for humans, feed for
transplanted rice. They are not subjected to stress
animals, and fertilizer for plants.
like being pulled from the soil of the nursery and do
not need to reproduce fine rootlets. ❖ Products like agar-agar and alginates, iodine which
are of commercial value, are extracted from
❖ It offers certain other advantages viz., it saves
seaweeds.
labour, requires less water, less drudgery, low
production cost, better soil physical conditions for ❖ By the biodegradation of seaweeds, methane-like
following crops and less methane emission, provides economically important gases can be produced in
better option to be the best fit in different cropping large quantities.
systems. ❖ Extracts of some seaweed species show antibacterial
❖ Statement 2 is correct and 3 is incorrect: activity.
However, despite the obvious upsides, DSR has the ❖ Seaweeds are used as a drug for goiter treatment,
following disadvantages: intestinal and stomach disorders.
❖ There is greater crop-weed competition because rice ❖ Seaweeds are also used as the potential indicators of
plants and weeds are of similar age. And hence the pollution in coastal ecosystems, particularly heavy
critical period of crop weed competition is crucial
24

metal pollution due to their ability to bind and disorders. The pharmacological activity of rauvolfia
accumulate metals strongly. is due to the presence of several alkaloids of which
reserpine is the most important, which is used for its
sedative action in mild anxiety states and chronic
Q.57) Ans: B psychoses.
Exp: ❖ It has a depressant action on the central nervous
system that produces sedation and lower blood
❖ Statement 1 is incorrect: Mixed farming is a type
pressure.
of farming that involves both the growing of crops
and the raising of livestock. Equal emphasis is laid ❖ The root extracts are used for treating intestinal
on crop cultivation and animal husbandry. Animals disorders, particularly diarrhoea and dysentery and
like cattle, sheep, pigs, and poultry provide the main also anthelmintic. It is used for the treatment of
income along with crops. Mixed farming is cholera, colic and fever. The juice of the leaves is
characterized by high capital expenditure on Agri used as a remedy for opacity of the cornea. The total
inputs such as farm machinery and building, root extracts exhibit a variety of effects, viz.,
extensive use of chemical fertilizers and green sedation, hypertension, brodyeardia, myosis, ptosis,
manures, and also the skill and expertise of the tremors, which are typical of reserpine.
farmers. This form of agriculture is found in the
highly developed parts of the world, e.g., North-
western Europe, Eastern North America, parts of Q.59) Ans: C
Eurasia, and the temperate latitudes of Southern
Exp:
continents.
❖ Statement 1 is incorrect: Growing two or more
❖ Statement 2 is correct: Mixed farms are moderate
crops simultaneously with distinct row arrangement
in size and usually, the crops associated with them
on the same field at the same time. There is no
are wheat, barley, oats, rye, maize, fodder, and root
competition between main and inter crop (subsidiary
crops. Fodder crops are an important component of
crop).
mixed farming. Crop rotation and intercropping play
an important role in maintaining soil fertility. ❖ Base crop: primary crop which is planted/ shown at
its optimum sole crop population in an intercropping
situation.
Q.58) Ans: B
❖ Intercrop: This is a second crop planted in between
Exp: rows of base crop with a view to obtain extra yields
with intercrop without compromise in the main crop
❖ Statement 1 is incorrect: Sarpagandha or Rauvolfia
yields.
serpentina is a medicinal plant found in the sub-
Himalayan tract from Punjab eastwards to Nepal, ❖ The main objective of intercropping is to utilize the
Sikkim, Assam, Eastern & Western Ghats, parts of space between rows of main crop and to produce
Central India and in the Andamans. more grain per unit area.

❖ Statement 2 is correct: Rauvolfia roots are of ❖ Advantages of Intercropping:


immense medicinal value and have steady demand.
It is used for treating various central nervous system
25

❖ Statement 2 is correct: Successful intercropping ❖ Statement 2 is incorrect: Zero Tillage: In Zero


gives higher equivalent yields (yield of base crop + Tillage a new crop is planted in the residues of the
yield of intercrop), higher cropping intensity. previous crop without any prior soil tillage or seed
bed preparation and it is possible when all the weeds
❖ Better use of growth resources including light,
are controlled by the use of herbicides.
nutrients and water.
❖ Advantages of Zero tillage are as following:
❖ Suppression of weeds.
❖ Zero tilled soils are homogenous in structure with a
❖ Yield & stability - even if one crop fails due to
greater number of earthworms
unforeseen situations, another crop will yield and
provide some secured income. ❖ Organic matter content increases due to less
mineralization
❖ Reduced pest and disease incidences
❖ Surface runoff is reduced due to presence of mulch
❖ Improvement of soil health and agro-eco system

❖ Statement 3 is correct: Alley cropping is a system


in which food crops are grown in alleys formed by Q.61) Ans: A
hedge rows of trees or shrubs. The essential feature
Exp:
of the system is that hedgerows are cut back at
❖ Statement 1 is incorrect: Moho’s discontinuity
planting and kept pruned during cropping to prevent
exists between the mantle and crust interface.
shading and to reduce competition with food crops.
Discontinuity is an interface where the seismic
Example Subabul raised at 6 m row spacing.
waves show change in their velocity.
❖ Statement 2 is incorrect: Gutenberg’s discontinuity
Q.60) Ans: A exists between the mantle and the core interface.
❖ Statement 3 is correct: Asthenosphere exists in the
Exp: upper part of the mantle. This layer houses the
❖ Statement 1 is correct: Tillage: Tillage is the magma that finds its way out during a volcanic
mechanical manipulation of soil with tools and eruption. It is a weak viscous layer extending upto
implements for obtaining conditions ideal for seed 400 km.
germination, seedling establishment and growth of ❖ Statement 4 is incorrect: Lithosphere is the layer
crops. comprising the core and upper most solid part of the
mantle. It lies between 10-200 km from the surface
❖ Advantages of Tillage are as following: of the earth.
❖ Greater volume of soil may be obtained for
cultivation of crops.
Q.62) Ans: A
❖ Excess water may percolate downward to recharge
Exp:
the permanent water table.
❖ Permaculture: Permaculture is defined as a design
❖ Reduce runoff and soil erosion
system for creating sustainable human
❖ Roots of crop plants can penetrate deeper to extract environments. It uses ecology as the basis for
❖ Moisture from the water table designing integrated systems of food production,
housing, appropriate technology, and community
26

development. Permaculture is built upon an ethic of ✓ Distribution: Odisha, Jharkhand,


caring for the earth and interacting with the Chhattisgarh, Andhra Pradesh, Karnataka,
environment in mutually beneficial ways. Maharashtra and Goa

❖ Characteristics of Permaculture: ✓ Formed in the Cuddapah rocks and


Dharwad System
❖ It is one of the most holistic, integrated systems
analysis and design methodologies found in the ➢ Magnetite
world.
✓ magnetite actually has higher iron content
❖ Statement 1 is correct: It can be applied to create (70-75%) than the mineral hematite.
productive ecosystems from the human use However, while hematite ore generally
standpoint or to help degraded ecosystems recover contains large concentrations of hematite,
health and wildness. magnetite ore generally holds low
concentrations of magnetite.
❖ Statement 3 is correct: It can be applied in any
ecosystem, no matter how degraded. It values and ✓ Hence the ore needs to be beneficiated
validates traditional knowledge and experience. (treat to improve its properties) for
magnetite recovery.
❖ Incorporates sustainable agriculture practices and
land management techniques and strategies from ✓ Taconite and Lodestone are forms of
around the world. Magnetite

❖ It is a bridge between traditional cultures and ✓ Distribution: Odisha, Jharkhand,


emergent earth-tuned cultures. Chhattisgarh, Andhra Pradesh,

❖ Statement 2 is correct: It promotes organic ➢ Limonite


agriculture, which does not use pesticides.
✓ inferior iron ore that is yellowish in colour
❖ Statement 4 is incorrect: It aims to maximize with 40 to 60 per cent iron content
symbiotic and synergistic relationships between site
✓ Distribution: Raniganj, Garhwal
components.
(uttarakhand) and Kangra Valley
❖ Its design is site specific, client specific, and culture (himachal)
specific.
✓ Siderite

➢ It is iron carbonate ore of inferior quality with


Q.63) Ans: C less than 40 per cent iron content.

Exp: ➢ Invariably has many impurities and hence


mining, in many places, is economically
❖ Option C is correct: Iron ore: Iron is taken out
unviable.
from mines in the form of iron ore. There are
different types of iron ore containing varying ➢ Self-Fluxing due to natural presence of
percentage of pure iron: Limestone (Separates impurities on its own,
when in liquid form, without addition of extra
➢ Haematite
flux agents).
✓ 60 to 70 percent pure iron
27

Extra Edge by ONLYIAS:

Iron ore details:

❖ India has large iron ore reserves. It occurs in


various geological formations but major
economic deposits are found in volcano-
sedimentary Banded Iron Formation (BIF) from
the Precambrian age.

❖ India's leading state that produces iron ore is


Odisha. It accounts for more than 55% of the
total production followed by Chhattisgarh
producing almost 17%. This is followed by
Karnataka and Jharkhand producing 14% and Q.65) Ans: B
11% respectively.
Exp:
❖ In India, the value of metallic minerals in 2018-
19 was at INR 64,044 crores. Among these the ❖ Options 1,2,3 and 5 are correct:
principal metallic minerals, iron ore contributed ❖ Metallic minerals are the minerals containing one or
more than 45000 crore rupees. more metals. These usually occur as mineral
deposits and are a great heat and electricity
conductor. They can be ferrous or non-ferrous.
Q.64) Ans: D
Ferrous Non-Ferrous
Exp:

❖ Option D is correct: Mines in ODISHA Chromite, Gold, Silver, Copper, Lead,


manganes bauxite, Magnesium, Tin.
❖ As can be seen in the map, Odisha has large reserves
e, Pyrites,
of almost all kinds of minerals that are commercially
Tungsten,
exploited for various industries in India.
Nickel,
❖ Odisha has the largest reserves of bauxite, iron ore Cobalt.
and Chromite in India.
❖ Nonmetallic minerals lack metallic characteristics
like good electric and thermal conductivity, luster,
rigor, and malleability; they are, however, essential
for many industries.

❖ Examples: Limestone, nitrate, Potash, mica,


gypsum, Dolomite.

Q.66) Ans: D
28

Exp: ❖ Krishna-Godavari

❖ Option D is correct: Oil Shales are usually fine- ❖ Kaveri


grained sedimentary rocks containing relatively
❖ Indo-Gangetic plain
large amounts of organic matter from which
significant quantities of shale oil and combustible
gas can be extracted by destructive distillation.
Q.67) Ans: B
❖ Shale gas is a mixture of a lot of methane, little
Exp:
ethane, propane, & butane, very little carbon
dioxide, nitrogen, and hydrogen sulphide. ❖ Option B is correct: Hajira-Bijapur-Jagdishpur
Gas Pipeline (HVJ Pipeline): The project was
❖ Sandstone rocks often have high permeability,
started in 1986 after the incorporation of GAIL
which means that the tiny pores within the rock are
(India) Limited to supply gas to the fertilizer plants
well connected and gas can flow easily through the
located in the state of Uttar Pradesh. The first phase
rock. In contrast, shale rocks where gas is trapped as
of the project consisting of a non-branched 1,750-
a continuous accumulation throughout a large area
kilometer grid was commissioned in 1987. Later on,
usually have very low permeability, making gas
the system was expanded with additional branches
production more complex and costly.
to supply gas for industrial and domestic use in the
states of Rajasthan, Haryana and NCT, which
increased the total grid length to 3,474 km.

❖ Shale gas can emerge as an important new source of


energy in the country. India has several Shale
Formations which seem to hold shale gas Q.68) Ans: B
❖ The Ministry of Petroleum and Natural Gas Exp:
(MoPNG) has identified six basins as potentially
❖ Statement 1 is incorrect: Under Entry 24 of the
shale gas bearing. These are
Union List of the Seventh Schedule of the
❖ Cambay Constitution, the central government can make laws
❖ Assam-Arakan on shipping and navigation on inland waterways
which are classified as national waterways by
❖ Gondwana Parliament by law.
29

❖ Thus, the 111 inland waterways (including 5


National Waterways declared earlier) were declared
as ‘National Waterways’ under the National
Waterways Act, 2016.

❖ The Act repeals the five Acts that declare the


existing national waterways. These five national
waterways are now covered under the Bill.

❖ Statement 3 is incorrect: As of now National


Waterway (NW)-1 Ganga- Bhagirathi-Hooghly
River system from Allahabad to Haldia), NW-2
(River Brahmaputra from Dhubri to Sadiya) in
Assam, NW-3 (West Coast Canal from Kottapuram
to Kollam along with Udyogamandal and
Champakara Canals) in Kerala, have already been
developed with fairway navigational aids, jetties and
terminals with mechanized equipment handling
facilities for loading and unloading of cargo. These
NWs are operational and vessels are plying on
them.

❖ In addition, NW-10 (river Amba), NW-68 (river


❖ Statement 2 is correct: The National Waterway-4 Mandovi), NW-73 (river Narmada), NW-83
connects the Puducherry to Godavari River system (Rajpuri Creek), NW-85 (Revadanda Creek -
via Krishna River system Kakinada and Kundalika River System), NW-91 (Shastri river–
Buckingham Canals. It covers 6 states as shown Jaigad creek system), NW- 97 (Sunderbans
here. Waterways), NW-100 (river Tapi) and NW-111
❖ The NW-1 (Ganga-Bhagirathi-Hooghly River (river Zuari) are also operational.
System (Haldia - Allahabad)) on the other hand ❖ On the other hand, Feasibility studies have been
covers 4 states only. completed for 106 new NWs and based on the
outcome of feasibility studies and Detailed Project
Reports (DPRs), 20 new NWs have been found
technically feasible for development of shipping and
navigation in addition to 5 existing NWs by the
IWAI.

Q.69) Ans: B

Exp:
30

❖ The Indian Railway was introduced in 1853, when a generator. There are three types of hydropower
line was constructed from Bombay to Thane facilities: impoundment, diversion, and pumped
covering a distance of 34 km. Indian Railways is the storage. Some hydropower plants use dams and
largest government undertaking in the country. On some do not.
the basis of the width of track of the Indian
❖ Statement 1 is incorrect: Impoundment of station:
Railways, three categories have been made:
The most common type of hydroelectric power plant
❖ Statement 1 is incorrect: Broad gauge: The is an impoundment facility. An impoundment
distance between rails in broad gauge is 1.676 metre. facility, typically a large hydropower system, uses a
The total length of broad-gauge lines was 60510 km dam to store river water in a reservoir. Water
in March 2016. released from the reservoir flows through a turbine,
spinning it, which in turn activates a generator to
❖ Metre gauge: The distance between rails is one
produce electricity.
metre. Its total length was 3880 km in March 2016.
❖ Statement 2 is incorrect: Diversion power station:
❖ Statements 2 and 3 are correct: Narrow gauge:
A diversion, sometimes called run-of-river facility,
The distance between the rails in this case is 0.762
channels a portion of a river through a canal or
metre or 0.610 metre. The total length of the narrow
penstock and then to flow through a turbine,
gauge was 2297 km in March 2016. It is generally
spinning it, which in turn acti-vates a generator to
confined to hilly areas.
produce electricity. It may not require the use of a
❖ Indian Railways has launched an extensive dam.
programme to convert the metre and narrow gauges
❖ Statement 3 is correct: Pumped storage: It works
to broad gauge. Moreover, steam engines have been
like a battery, storing the electricity generated by
replaced by diesel and electric engines. This step has
other power sources like solar, wind, and nuclear for
increased the speed, as well as, the haulage capacity.
later use. When the demand for electricity is low, a
The replacement of steam engines run by coal has
pumped storage facility stores energy by pumping
also improved the environment of the stations.
water from a lower reservoir to an upper reservoir.
❖ The length of the Indian Railways network was During periods of high electrical demand, the water
66,030 km as on 31 March 2015. It's very large size is released back to the lower reservoir and turns a
puts a lot of pressure on a centralized railway turbine, generating electricity.
management system. Thus, in India, the railway
system has been divided into 16 zones.
Q.71) Ans: B

Exp:
Q.70) Ans: C
❖ Fundamental rights are certain claims which are
Exp:
fundamental to the all-round development of an
❖ Types of hydro power stations: Hydraulic power individual. These rights are recognized by the state
can be captured when water flows downward from a and guaranteed by the Constitution itself. These
higher level to a lower level which is then used to claims are termed as Fundamental Rights (FRs).
turn the turbine, thereby converting the kinetic Fundamental Rights prevent arbitrary rule of the
energy of water into mechanical energy to drive the state by promoting the ideal of political democracy,
31

❖ Fundamental rights available to both citizens and ➢ Article 29 and 30: cultural and educational
foreigners except enemy aliens rights.

➢ Article 14 - Equality before the law and equal


protection of laws.
Q.72) Ans: B
➢ Article 20 - pertains to Protection in respect of
Exp:
conviction for offences.
❖ Statement 1 is incorrect: Section 144 of the
➢ Article 21 - signifies Protection of life and
Criminal Procedure Code (CrPC) of 1973 generally
personal liberty.
prohibits public gathering. It authorises the
➢ Article 21A - Right to elementary education. Executive Magistrate of any state or territory to
issue an order to prohibit the assembly of four or
➢ Article 22 - Protection against arrest and
more people in an area.
detention in certain cases.
❖ According to the law, every member of such
➢ Article 23 - Prohibition of traffic in human
'unlawful assembly' can be supense for engaging in
beings and forced labour.
rioting. The maximum punishment for violation of
➢ Article 24 - Prohibition of employment of section 144 is three years. Moreover, obstructing
children in factories etc. police from breaking up an unlawful assembly is a
punishable offence as well. Section 144 also restricts
➢ Article 25 - Freedom of conscience and free
carrying any sort of weapon in that area where
profession, practice and propagation of religion.
section 144 has been imposed and people can be
➢ Article 26 - Freedom to manage religious detained for violating it.
affairs.
❖ Statement 2 is correct: Section 144 is imposed in
➢ Article 27 – Freedom from payment of taxes urgent cases of nuisance or apprehended danger of
for promotion of any religion. some event that has the potential to cause trouble or
➢ Article 28 – Freedom from attending religious damage to human life or property. No order under
instruction or worship in certain educational Section 144 shall remain in force for more than two
institutions. months but the state government can extend the
validity for two months and maximum up to six
❖ There are some Fundamental rights which are months. It can be withdrawn at any point of time if
only available to citizens of India are: the situation becomes normal.
➢ Article 15: Right against discrimination on the ❖ Section 144 was used for the first time in 1861 by
grounds of religion, race, caste, sex or place of the British Raj, and thereafter became an important
birth; tool to stop all nationalist protests during the
➢ Article 16: right to equality of opportunity in Freedom Struggle. However, the use of the section
matter of public employment; in Independent India remains controversial as very
little has changed.
➢ Article 19: freedom of speech and expression,
assembly, association, movement, residence ❖ Statement 3 is correct: Section 144 is generally
and profession; prohibitory in nature. It restricts from public
gathering but doesn't bar it all together. A curfew, on
32

the other hand, orders people to stay indoors for a ❖ Statement 3 is correct: One of the duties and
specific period of time. So, the authorities can functions performed by National Commission for
impose curfew for a certain period of time Scheduled Tribe is present to the President, annually
(However, the authorities can also extend the curfew and at such other times as the Commission may
if need be). One also needs prior approval from the deem fit, reports upon the working of those
local police for moving out during curfew. safeguards. Other functions like Commission
inquires specific complaints with respect to the
deprivation of rights and safeguards of the STs and
Q.73) Ans: C discharge such other functions in relation to the
protection, welfare and development and
Exp:
advancement of the Scheduled Tribes as the
❖ Statement 1 is correct: National Commission for President may be subject to the provisions of any
Scheduled Tribe is a constitutional body that was set law made by Parliament by rule specified.
up with effect from 19th February 2004 by
amending Article 338 and by inserting a new article
338A in the Constitution through the 89th Q.74) Ans: C
Constitution Amendment Act, 2003. Article 338A
Exp:
inter-alia gives powers to the NCST to oversee the
implementation of various safeguards provided to ❖ Statement 1 is correct: Reasonable
STs under the Constitution or under any other law accommodation’ is a principle that promotes
for time being in force or under any other order to equality, enables the grant of positive rights and
the Government and to evaluate the working of such prevents discrimination based on disability, health
safeguards. condition or personal belief.

❖ Statement 2 is incorrect: The Constitution lays ❖ Its use is primarily in the disability rights sector. It
down the composition of NCST, consisting of a captures the positive obligation of the State and
Chairperson, a Vice-Chairperson and 3 other private parties to provide additional support to
Members who are appointed by the President by persons with disabilities to facilitate their full and
warrant under his hand and seal. effective participation in society.

❖ At least one member should be a woman. ❖ Statement 2 is incorrect: The International Labour
Organisation in 2016, came out with a practical
❖ The Chairperson, the Vice-Chairperson and the
guide on promoting diversity and inclusion through
other Members hold office for a term of 3 years.
workplace adjustments. The need for workplace
❖ The members are not eligible for appointment for accommodation can arise in various situations, but
more than two terms. four categories of workers were chosen for the
guide:
❖ The Chairperson has been given the rank of Union
Cabinet Ministers, the Vice Chairperson has the rank ➢ Workers with disabilities,
of a
➢ Workers living with HIV and AIDS,
❖ Minister of State and other Members have the rank
➢ Pregnant workers and those with family
of a Secretary to the Government of India.
responsibilities, and
33

➢ Workers who hold a particular religion or ❖ Statement 2 is correct: The Ministry of Home
belief. Affairs (MHA) has amended the Foreigners
(Tribunals) Order, 1964, and has empowered district
❖ These categories of workers come across different
magistrates in all States and Union Territories to set
kinds of barriers at work. These can result either in
up tribunals (quasi-judicial bodies) to decide
loss of employment or in lack of access to
whether a person staying illegally in India is a
employment. The provision of reasonable
foreigner or not. Earlier, the powers to constitute
accommodation plays a major role in addressing
tribunals were vested only with the Centre.
these barriers and thus contributes to greater
workplace equality, diversity and inclusion. ❖ Statement 3 is incorrect: The amended order
(Foreigners (Tribunal) Order, 2019) also empowers
❖ Statement 3 is correct: In India, the Rights of
individuals to approach the Tribunals. Earlier, only
People with Disabilities Act, 2016, defines
the State administration could move the Tribunal
‘reasonable accommodation’ as “necessary and
against a suspect. A declared foreigner, or DF, is a
appropriate modification and adjustments, without
person marked by Foreigners’ Tribunal (FT) for
imposing a disproportionate or undue burden in a
allegedly failing to prove their citizenship after the
particular case, to ensure to persons with disabilities
State police’s Border wing marks him or her as an
the enjoyment or exercise of rights equally with
illegal immigrant.
others”. The Section 2(h) defines ‘discrimination’ as
'denial of reasonable accommodation’.

❖ Vikash Kumar v. UPSC (2021): The Supreme court Q.76) Ans: D


held that benchmark disability, that is a specified
Exp:
disability to the quota of 40 percent, is related only
to special reservation for the disabled in ❖ Recently, the National Commission for Women
employment, but it should not be a restriction for (NCW) in collaboration with Delhi State Legal
other kinds of accommodation. Services Authority (DSLSA) has launched a Legal
Aid Clinic. It will act as a single-window facility for
resolving the grievances of women by offering them
Q.75) Ans: A free legal assistance.

Exp: ❖ The legal aid clinic will operate out of the NCW
office in New Delhi. Under the clinic, counselling
❖ Statement 1 is correct: Foreigners’ Tribunals are
will be provided for walk-in complainants, women
quasi-judicial bodies established as per the
in distress, advice and information on various
Foreigners’ Tribunal Order, 1964 and the
schemes of the National Legal Services Authority
Foreigners’ Act, 1946. Composition of Foreigners
(NALSA), free legal aid, hearings in matrimonial
Tribunal includes Advocates not below the age of 35
cases and other complaints registered with the
years of age with at least 7 years of practice (or)
Commission.
Retired Judicial Officers from the Assam Judicial
Service (or) Retired IAS of ACS Officers (not below ❖ National Commission for Women:
the rank of Secretary/Addl. Secretary) having
❖ It was set up as a statutory body in January 1992
experience in quasi-judicial works.
under the National Commission for Women Act,
1990. Its mission is to strive towards enabling
34

women to achieve equality and equal participation in India which coordinates investigations on behalf of
all spheres of life by securing her due rights and Interpol Member countries.
entitlements through suitable policy formulation,
legislative measures, etc.
Q.78) Ans: C
❖ Functions of the National commission for women
Exp:
❖ Review the constitutional and legal safeguards for
women. ❖ The Ministry of Parliamentary Affairs (MoPA) has
developed a “National eVidhan Application
❖ Recommend remedial legislative measures.
(NeVA)”, a Mission Mode Project for Digital
❖ Facilitate redressal of grievances. Legislatures to make the functioning of all
Legislative Houses in the country paperless.
❖ Advise the Government on all policy matters
National eVidhan Application(NeVA) is a Mission
affecting women.
Mode Project (MMP) included in the Digital India
Programme. To make the functioning of all
Legislative Houses in the country paperless on the
Q.77) Ans: B
theme of ‘One Nation –One Application’.
Exp:
❖ To transform all State Legislatures into ‘DIGITAL
❖ Statement 1 is incorrect: Unlike the National HOUSES’ so as to enable them to transact entire
Investigation Agency(NIA), CBI cannot take suo Government Business on digital platforms including
motu cognizance of a case in a state — whether in a information exchange with the State Government
matter of corruption involving government officials Departments in digital mode. The application has
of the Centre and PSU staff or an incident of violent also enabled provisions for onboarding the two
crime. In order to take up corruption cases involving Houses of Parliament.
central government staff, it either needs general
❖ The funding for e-Vidhan is provided by the MoPA.
consent of the state government or specific consent
It is on the pattern of Central Sponsored Scheme, i.e.
on a case-to-case basis.
60:40 for States; 90:10 for North East & hilly States
❖ For all other cases, whether involving corruption in and 100% for UTs. In 2021, Bihar Legislative
the state government or an incident of crime, the Council became the first House in the country to
state has to request an investigation by the CBI, and transit to NeVA platform completely and conducted
the Centre has to agree to the same. In case the state the Winter Session, 2021 on the NeVA platform in
does not make such a request, the CBI can take over paperless mode.
a case based on the orders of the High Court
Concerned or the Supreme Court.
Q.79) Ans: D
❖ Statement 2 is correct: Central Bureau of
Investigation (CBI) is the premier investigating Exp:
police agency in India. It functions under the
❖ Statement 1 is correct: The Governor’s
superintendence of the Deptt. of Personnel, Ministry
appointment, his powers and everything related to
of Personnel, Pension & Public Grievances,
the office of Governor have been discussed under
Government of India - which falls under the prime
Article 153 to Article 162 of the Indian Constitution.
minister’s office. It is also the nodal police agency in
35

Governor is the constitutional head of the state, Assembly for a floor test on the recommendation of
bound by the advice of his council of ministers. He the Cabinet. A Governor cannot employ his
functions as a vital link between the Union “discretion”, and should strictly abide by the “aid
Government and the State Government. and advice” of the Cabinet to summon the Assembly
for a floor test. Governor’s discretionary powers are
❖ Statement 2 is incorrect: The governor has two
limited to specified areas like giving assent or
categories of discretion, Constitutional Discretion
withholding/referring a Bill to the President or
and discretion mentioned in the Constitution.
appointment of a Chief Minister or dismissal of a
❖ Situational Discretion is a hidden discretion derived government which has lost of confidence but refuses
from the exigencies of a prevailing political to quit, etc.
situation. Appointment of a chief minister when no
party has a clear majority in the state legislative
assembly or when the current chief minister dies Q.80) Ans: B
unexpectedly and there is no obvious successor, is a
Exp:
situational discretion. Other situational discretion
includes Dismissal of the council of ministers when ❖ Concerns with the current practice of appointing
it is unable to demonstrate the confidence of the outsiders as Secretary-Generals:
state legislative assembly. Dismissal of state
❖ Against the principle of separation of powers:
legislative assembly when the council of ministers
Appointing serving or retired civil servants to the
lose their majority.
post of Secretary-General amounts to the breach of
❖ According to the Constitution the governor has the principle of separation of power as envisaged
constitutional discretion in the following cases: under the Indian Constitution and also upheld by the
judiciary as a basic structure of the Constitution.
❖ Reservation of a bill passed by legislative assembly
Article 98 of the Constitution by providing for two
for the consideration of the President.
separate secretariats emphasizes the principle that
❖ Recommendation for the imposition of the the secretariats should be independent of the
President’s Rule in the concerned state. executive government. A separate and independent
secretariat marks a feature of a functioning
❖ When he is exercising his functions as the
parliamentary democracy. Appointing civil servants
administrator of an adjoining union territory.
to the post of Secretary-General, could undermine
❖ Determining the amount payable by the Government the independence of the Secretariat.
of Assam, Meghalaya, Tripura and Mizoram to an
❖ Lack of knowledge of the functioning of the
autonomous Tribal District Council as royalty
legislature: The Secretary-General is entrusted with
accruing from licences for mineral exploration.
a wide range of duties and functions which
❖ Seeking information from the chief minister with necessitate vast knowledge and rich experience of
regards to the administrative matters of the state. Parliamentary procedures, practices and precedents.
Most of the civil servants may lack this expertise.
❖ Statement 3 is correct: The Constitution Bench
judgement of the Supreme Court in Nabam Rebia ❖ Impact Legislatures' role of holding the executive
versus Deputy Speaker on July 13, 2016, held that a accountable: In a parliamentary polity like India,
Governor is bound to convene a meeting of the apart from passing legislations, the Parliament is
36

also expected to scrutinise the executive’s Marmara. It is the only passage through which the
administrative behaviour. A strong Parliament Black Sea ports can access the Mediterranean and
means a more accountable executive. Appointing beyond.
former bureaucrats to head important legislature
❖ Montreux Convention is the international agreement
bodies like the Secretariat could lead to possible
was signed by Australia, Bulgaria, France, Greece,
conflict of interests.
Japan, Romania, Yugoslavia, the United Kingdom,
the Soviet Union and Turkey and has been in effect
since November 1936 is Regarding the Regime of
Q.81) Ans: C
the Straits is an international agreement governing
Exp: the Bosporus and Dardanelles Straits in Turkey.
❖ Statement 1 is correct: The Conduct of Election ❖ This pact gives authority to Turkey to have control
Rules, 1961 was amended in 2016 to allow service over both the Black Sea Straits. In the event of a
voters to use the ETPBS. Under this system, postal war, the pact gives Ankara the right to regulate the
ballots are sent electronically to registered service transit of naval warships and to block the straits to
voters. The service voter can then download the warships belonging to the countries involved in the
ETPB (along with a declaration form and covers), conflict. Article 19 of the treaty contains an
register their mandate on the ballot and send it to the exception for the countries on the Black Sea that can
returning officer of the constituency via ordinary effectively undermine Turkey’s power in blocking
mail. The post will include an attested declaration the Russian warships entering or exiting the Black
form (after being signed by the voter in the presence Sea.
of an appointed senior officer who will attest it).
❖ The Article 19 says, “Vessels of war belonging to
❖ Statement 2 is correct: through the Representation belligerent powers, whether they are Black Sea
of the People (Amendment) Act, 2010, eligible NRIs Powers or not, which have become separated from
who had stayed abroad beyond six months were their bases, may return thereto.” That means
allowed to vote, but only in person at the polling warships can return to their actual bases through the
station where they have been enrolled as an overseas passage and Turkey cannot prevent it. The term
elector. An NRI can vote in the constituency in applies to Russian fleets also currently in the Black
his/her place of residence, as mentioned in the Sea that belong to a base in the Mediterranean or
passport, is located. He/She can only vote in person Baltic Sea. Russia is free to take them out of the
and will have to produce her passport in original at Black Sea.
the polling station for establishing identity.

Q.83) Ans: C
Q.82) Ans: D
Exp:
Exp:
❖ PM DAKSH (Pradhan Mantri Dakshta Aur Kushalta
❖ Option D is correct: Black sea strait also known as Sampann Hitgrahi) is being implemented from the
the Turkish Straits, the Black Sea Straits are the year 2020-21. Under this, eligible target groups are
Bosporus and Dardanelles strait and connect the provided with the skill development training
Aegean Sea and the Black Sea via the Sea of programmes on Short Term Training Program; and
37

Up-Skilling/Reskilling; Entrepreneurship ❖ Statement 2 is correct: In the case of Zahoor


Development Programme, and Long Term Training Ahmed Shah Watali, the Supreme Court in 2019
Programme. confirmed that courts must accept the state’s case
without examining its merits. However, courts have
❖ These training programmes are being implemented
since read this provision differently, emphasising the
through the government training institutions, sector
right to a speedy trial and raising the bar for the state
skill councils which have been constituted by the
to book an individual under UAPA.
Ministry of Skill Development and
Entrepreneurship, and other credible institutions.
Marginalised persons of SC, OBC , EBC, Denotified
Q.85) Ans: B
tribes, Sanitation workers including waste pickers,
manual scavengers, transgenders and other similar Exp:
categories will be Eligible for this scheme.
❖ Statement 1 is incorrect: The Constitution does not
❖ The program is implemented by the 3 define the criteria for recognition of Scheduled
Corporations under the Ministries as: Tribes (ST) and hence the definition contained in
1931 Census was used in the initial years after
➢ National Scheduled Castes Finance and
independence.
Development Corporation (NSFDC),
❖ However, Article 366(25) of the Constitution only
➢ National Backward Classes Finance and
provides a process to define Scheduled Tribes (ST):
Development Corporation (NBCFDC),
“Scheduled Tribes means such tribes or tribal
➢ National Safai Karamcharis Finance and communities or parts of or groups within such tribes
Development Corporation (NSKFDC). or tribal communities as are deemed under Article
342 to be Scheduled Tribes for the purposes of this
Constitution.
Q.84) Ans: D
❖ Statement 2 is incorrect: The 5th Schedule lays out
Exp: provision for Administration and Control of
Scheduled Areas and Scheduled Tribes in states
❖ Statement 1 is correct: The major problem with the
other than Assam, Meghalaya, Tripura and Mizoram
UAPA lies in its Section 43(D)(5), which prevents
(AMTM).
the release of any accused person on bail if, police
have filed the chargesheet that there are reasonable ❖ The 6th Schedule deals with the administration of
grounds for believing that the accusation against the tribal areas in Assam, Meghalaya, Tripura and
such person is prima facie true. Mizoram. There are over 705 tribes which have been
notified. The largest number of tribal communities
❖ The effect of Section 43(D)(5) is that once the police
are found in Odisha.
elect to charge an individual under the UAPA, it
becomes extremely difficult for bail to be granted. ❖ Statement 3 is incorrect: Darlong is a tribal
Bail is a safeguard and guarantee of the community of Tripura, which has a population of
constitutional right to liberty. This provision leaves 11,000. The community has a high prevalence of
very little room for judicial reasoning, and makes education and cultural activity and its members of
the grant of bail virtually impossible under UAPA. the community serve in senior positions in the local
administration. For example, a tribal musicologist
38

and Rosem (a tribal instrument) maestro Thanga in the travel, tourism and hospitality sectors.A total
Darlong was awarded the prestigious Padma Shri a of 82 international delegates are also participating in
few years ago for his contributions to culture. the WTD Celebration.

Q.86) Ans: d Q.87) Ans: c

Exp: Exp:

❖ All statements are correct: United Nations World ❖ Statement 1 is correct: The first Raisina Dialogue
Tourism Organisation (UNWTO) is the United was held from March 1st March 2016. Over 100
Nations specialized agency assinged with the speakers from over 35 countries attended to speak
promotion of responsible, sustainable and on the theme, “Asia: Regional and Global
universally accessible tourism; encourages the Connectivity”. The main focus of the 2016
implementation of the Global Code of Ethics for conference was on Asia’s physical, economic,
Tourism worldwide. India also joined UNWTO in human and digital connectivity. Raisina Dialogue is
1975. It is headquartered in Madrid, Spain. a multilateral conference committed to addressing
the most challenging issues facing the global
❖ UNWTO has chosen India as the host country to
community. Each year, global leaders in policy,
celebrate World Tourism Day 2019 on the
business, media and civil society are hosted in New
theme ‘Tourism and Jobs: A better future for
Delhi to discuss cooperation on a wide range of
all’. World Tourism Day Celebrated every year on
pertinent international policy matters
September 27 around the world.
❖ Statement 2 is incorrect: Raisina Dialogue is an
❖ The purpose of World Tourism Day (WTD) is to
annual geo-political event, organised by the Ministry
foster awareness among the international community
of External Affairs and Observer Research
on the importance of tourism and its social, cultural,
Foundation (ORF) and it is designed to explore
political and economic value and the event seeks to
prospects and opportunities for Asian integration as
address global challenges outlined in the United
well as Asia’s integration with the larger world and
Nations Millennium Development Goals and
predicated on India’s vital role in the Indian Ocean
highlight the contribution the tourism sector can
Region and how India along with its partners can
make in reaching these goals.
build a stable regional and world order. The
❖ The Ministry of Tourism, Indian government Dialogue is structured as a multi-stakeholder, cross-
annually presents National Tourism Awards to sectoral discussion, involving heads of state, cabinet
various segments of the travel, tourism and ministers and local government officials, as well as
hospitality industry. These awards are presented to major private sector executives, members of the
State Governments / Union Territories, classified media and academics. The Year 2022 Dialogue
hotels, tourist transport operators,heritage hotels, titled ‘Navigating the Alpha Century’ is structured
approved travel agents, tour operators, individuals as a multi-stakeholder, cross-sectoral discussion,
and other private organizations in recognition of involving heads of states, cabinet ministers and local
their performance in their respective fields. government officials as well as major private sector
executives, members of the media and academics.
❖ The National Tourism Awards have, over the years,
emerged as a megistrals recognition of achievements
39

❖ Statement 3 is correct: 7th edition Raisina biofuel. The use of crude palm oil in making
Dialogue 2022, based on the theme "Terranova- biodiesel is being branded as ‘green diesel’.
Impassioned, Impatient, Imperilled", will be
❖ Statement 3 is incorrect: India is the biggest
modelled along six thematic pillars –
importer of palm oil and meets half of its annual
❖ Rethinking Democracy: trade, technology and need for 8.3 million tons of palm oil from Indonesia
ideology. and China's palm oil imports are also expected to be
flat at 6.7 million tonnes in 2022. Indonesia is
❖ End of Multilateralism: a networked global order.
responsible for 60% of the global supply of palm oil.
❖ Water Caucuses: turbulent tides in the Indo-
❖ Palm oil is the world’s most widely used
Pacific.
vegetable oil with its global production in the year
❖ Communities Inc: first responders to health, 2020 being over 73 Million Tones (MT), according
development, and planet. to the United States Department of Agriculture
(USDA). It is estimated to be 77 MT for the current
❖ Achieving Green Transitions: common imperative,
year FY 2022-23. According to Reuters, palm oil
diverging realities.
makes up 40% of the global supply of the four most
❖ Samson vs Goliath: the persistent and relentless widely used edible oils: palm, soybean, rapeseed
technology wars. (canola), and sunflower oil.

Q.88) Ans: c Q.89) Ans: c


Exp: Exp:
❖ Statement 1 is correct: Indonesia, the world’s ❖ Statement 1 is correct: Desert national park is
biggest producer, exporter, and consumer of palm situated on the western border of India within
oil and Malaysia, second largest palm oil producer. Jaisalmer & Barmer in Rajasthan and it is
Indonesia and Malaysia together account for almost an excellent example of the Thar desert's ecosystem.
90% of the global palm oil production, with Spiny-tailed lizards, Krait, sandfish Desert monitors
Indonesia producing the largest quantity at over 45 and deadly vipers are the prominent members of the
million tonnes in 2021. wildlife family. In the winter season, the park hosts
❖ Statement 2 is correct: The production of palm oil an incredible variety of migratory raptors such
needs so much water but Indian states like Andhra Himalayan and Eurasian Griffon Vultures, Eastern
Pradesh depend on rainwater for production of palm Imperial Eagle, and the Saker Falcon.
oil and irrigation facilities are not good so have ❖ Statement 2 is correct: Rajasthan State Bird (Great
witnessed a water problem so switched to coconut Indian Bustard), State animal (Chinkara) and State
oil cultivation in Andhra Pradesh which is one of tree (Khejri) and State flowers (Rohida) are found
India's largest palm oil producers. Palm oil is an naturally in desert national park. The Desert
edible vegetable oil derived from the mesocarp National Park also has a collection of fossils of
(reddish pulp) of the fruit of the oil palms and it is animals and plants of 180 million years old and the
used as cooking oil, and in everything from Park is formed of undulating sand dunes, jagged
cosmetics, processed foods, cakes, chocolates, rocks, dense salt lake bottoms and inter-medial
spreads, soaps, shampoo, and cleaning products to
40

areas. It was recognised in 1980 as a UNESCO Exp:


World Heritage Site and in 1992 as a National Park.
❖ Statement 1 is correct: Chile is a country in the
western part of South America and it occupies a
long, narrow strip of land between the Andes to the
Q.90) Ans: d
east and the Pacific Ocean to the west.
Exp:
❖ Statement 2 is incorrect: It shares land borders
❖ Statement 1 is correct: International Finance with Peru to the north, Bolivia to the north-east,
Corporation( IFC) is a member of the World Bank Argentina to the east, and the Drake Passage in the
Group and it is an international financial institution far south and It is the world's southernmost country,
that offers investment, advisory, and asset the closest to Antarctica, that is geographically on
management services to encourage private sector the mainland. The country's capital and largest city
development in developing countries. It was is Santiago.
established in 1956 and headquartered in
Washington, D.C., United States.

❖ Statement 2 is correct: It was established as the


private sector arm of the World Bank Group
to advance economic development by investing in
strictly for-profit and commercial projects that
purport to reduce poverty and promote development.

❖ The IFC is owned and governed by its member


countries, but has its own executive leadership and
staff that conduct its normal business operations and
a corporation whose shareholders are member Q.92) Ans: c
governments that provide paid-in capital and which
Exp:
have the right to vote on its matters.
❖ Lithuania is a country in the Baltic region of
❖ Statement 3 is correct: Since 2009, the IFC has
Northern Europe. It shares land borders with Latvia
focused on a set of development goals that its
to the north, Belarus to the east and south, Poland to
projects are expected to target. Its goals are to
the south, and Kaliningrad Oblast of Russia to the
increase sustainable agriculture opportunities,
southwest. It has a maritime border with Sweden to
improve healthcare and education, increase access to
the west of the Baltic Sea and It is one of three
financing for microfinance and business clients,
Baltic states and lies on the eastern shore of the
advance infrastructure, help small businesses grow
Baltic Sea. Its capital and largest city is Vilnius.
revenues, and invest in climate health and advise
governments on building infrastructure and
partnerships to further support private sector
development.

Q.91) Ans: a
41

regional telecommunication organizations, which


may join ITU as non-voting Sector Members.

❖ Statement 3 is correct: The International


telecommunication Union works in three sectors:
Radiocommunication (ITU-R): This ensures
optimal, fair and rational use of the radio frequency
(RF) spectrum. Telecommunication Standardization
(ITU-T): It formulates recommendations for
standardizing telecommunication operations
worldwide. Telecommunication Development (ITU-
D): It assists countries in developing and
maintaining internal communication operations.
International telecommunication Union publishes
the Global Cybersecurity Index (GCI) and In
Q.93) Ans: b 2020 India ranked 10th in the index.

Exp:

❖ Statement 1 is correct: International Q.94) Ans: b


telecommunication Union is the United Nations Exp:
specialised agency for Information and
Communication Technologies – ICTs. Founded in ❖ Statement 1 is correct: United Nations Population
1865 to facilitate international connectivity in Funds (UNFPA): UNFPA formerly the UN Fund for
communications networks. It is Headquartered in Population Activities, is a UN agency aimed at
Geneva, Switzerland. ITU allocates global radio improving reproductive and maternal health
spectrum and satellite orbits, develops the technical worldwide; In 1969, the agency began operations as
standards which ensure networks and technologies the United Nations Fund for Population Activities
seamlessly interconnect, and strives to improve under the administration of the UN’s Development
access to ICTs to underserved communities Fund. It was placed under the authority of the
worldwide. UNGA in 1971. In 1987 its name was changed to
the United Nations Population Fund. It is
❖ Statement 2 is incorrect: ITU currently has a headquartered in New York.
membership of 193 countries and over 900 private-
sector entities and academic institutions. India got ❖ Statement 2 is correct: The work of the UNFPA
elected as a member of ITU Council for another 4- includes developing national healthcare strategies
year term - from 2019 to 2022. Since 1952 India has and protocols, increasing access to birth control, and
remained a regular member of ITU. Membership of leading campaigns against child marriage, gender-
ITU is open for UN members, which may join the based violence; fund assists countries in areas such
Union as Member States and to private as reproductive health, gender equality and human
organizations like carriers, equipment rights, safe motherhood, and adolescents and youth;
manufacturers, funding bodies, research and And Reliable access to modern contraceptives
development organizations and international and sufficient to benefit 20 million women a year.
42

❖ Statement 3 is correct: UNFPA has been assisting Ration Card (ONORC), vii) Janani Suraksha Yojana
the GOI since 1974 to provide family planning and and viii) Pradhan Mantri Matru Vandana Yojana.
health services, advance reproductive health and
❖ The socio-economic profiling will also help States if
rights and improve maternal health. UNFPA works
they deem fit to extend their State-specific welfare
with the government of India and partners to
schemes & benefits to the eligible PM SVANidhi
advocate for adolescents and youth’s rights and
beneficiaries and their families.
investments, including education, livelihood skills
and health, including sexual and reproductive health. ❖ Achievement of this scheme:

❖ 1st, a central database of Street vendors and their


families is created on various socio-economic
Q.95) Ans: b
indicators.
Exp:
❖ 2nd, a first of its kind inter-ministerial convergence
• Statement 1 is incorrect: The Ministry of Housing platform is established between various Central
and Urban Affairs (MoHUA) has launched Ministries to extend the safety net of welfare
‘SVANidhi se Samriddhi’ program in additional 126 schemes to street vendor households.
cities across fourteen States/ UTs and It is an
additional program of PMSVANidhi, launched on
January 4 in 2021 in 125 cities to map the socio- Q.96) Ans: b
economic profile of the PM SVANidhi beneficiaries Exp:
and their families.
❖ Statement 1 is incorrect: International Atomic
❖ Statement 2 is correct: Its purpose is to provide Energy Agency (IAEA) is an international
social security benefits to street vendors for their organization that seeks to promote the peaceful use
holistic development and socio-economic of nuclear energy and to prohibit its usage for any
upliftment. military purpose, including nuclear weapons. As the
❖ Statement 3 is correct: Its implementing partner major nuclear watchdog under the UN, the IAEA is
is Quality Council of India (QCI). Under the entrusted with the task of upholding the principles of
program, socio-economic profiling of PM the NPT of 1970;
SVANidhi beneficiaries and their families is ❖ It was established as an autonomous organisation on
conducted to assess their eligibility for 8 July 29, 1957, at the height of theCold War between
Government of India’s welfare schemes and the U.S. and the Soviet Union and Currently, it has
facilitate sanctions of eligible schemes. 171 members and India became a member in 1957.
❖ These Eight schemes include: i) Pradhan Mantri It was set up in 1957 and Headquarters in Vienna.
Jeevan Jyoti Bima Yojana, ii) PM Suraksha Bima ❖ It seeks to promote the peaceful use of nuclear
Yojana, ii) Pradhan Mantri Jan Dhan Yojana, iv) energy, and to inhibit its use for any military
Building and other Constructions Workers purpose, including nuclear weapons, establishing
(Regulation of Employment and Conditions of and administering safety guards to ensure that such
Service) Act (BOCW), v) Pradhan Mantri Shram research/development, etc. The IAEA is not used for
Yogi Maandhan Yojana, vi) National Food Security military purposes; Applying, under the Nuclear NPT
Act (NFSA)-portability benefit – One Nation One and other international treaties, mandatory
43

comprehensive safeguards in non-nuclear-weapon or sexual act or impersonation etc – social media


states parties to such treaties. platforms will be required to remove that content.

❖ Statement 2 is correct: India has recently become ❖ Statement 3 is incorrect: They also will have to
the 35th country to join IAEA Response and publish a monthly report about the number of
Assistance Network (RANET), a group of states complaints received and the status of redressal.
which offer assistance to mitigate the consequences There will be 4 levels of regulation for news
of nuclear or radiological emergencies. India publishers — self-regulation, a self-regulatory body,
actively participates in IAEA’s Advisory Groups headed by a retired judge or an eminent person,
and Technical Committees. oversight from the Information and Broadcasting
Ministry, including codes of practices and a
❖ India has close cooperation with the Agency’s
grievance committee and Social media companies
Programme of Action for Cancer Therapy (PACT)
with more than fifty lakh registered users will be
and would be donating under PACT its indigenously
considered ‘significant social media intermediaries’,
developed Cobalt teletherapy machine (Bhabhatron
as per the new norms.
II) to Sri Lanka and Namibia as a step towards
affordable treatment of Cancer. A similar machine In case of noncompliance:
was donated to Vietnam in 2008. AEA and its
❖ Social media giants such as Facebook, Twitter,
former Director General, Mohamed ElBaradei, were
Instagram and WhatsApp messenger could face a
jointly awarded the Nobel Peace Prize on 7 October
ban if they do not comply with the new Information
2005.
Technology rules.

❖ They also run the risk of losing their status as


Q.97) Ans: a “intermediaries” and may become liable for criminal
action if they do not comply with the revised
Exp:
regulations.
❖ Statement 1 is correct: IT Rules, 2021 mandates a
grievance redressal system for over the top (OTT)
and digital portals in the country. This is necessary Q.98) Ans: c
for the users of social media to raise their grievance
Exp:
against the misuse of social media and Significant
social media firms have to appoint a chief ❖ Statement 1 is correct: The Sarmat is a Russian
compliance officer and have a nodal contact person liquid-fueled intercontinental ballistic
who can be in touch with law enforcement agencies missile capable of nuclear charges. The RS-28
24/7. Sarmat, also called Satan-2, is reported to be able to
carry 10 or more warheads and decoys, and has the
❖ Statement 2 is incorrect: Social media platforms
capability of firing over either of the earth’s poles
will also have to name a grievance officer who shall
with a range of 11,000 to 18,000 km. This multiple
register the grievance within 24 hours and dispose of
independently targetable reentry vehicle (MIRV)-
it in 15 days. If there are complaints against the
equipped weapon can be deployed with 10 or more
dignity and respect of users, particularly women–
warheads on each missile.
about exposed private parts of individuals or nudity
44

❖ Statement 2 is correct: It is claimed that it has the Q.100) Ans: c


longest range of destruction of targets in the
Exp:
world. Sarmat is designed to elude anti-missile
defense systems with a short initial boost phase, ❖ Organization of the Petroleum Exporting
giving enemy surveillance systems a tiny window to Countries (OPEC): OPEC was founded in 1960 in
track. The name Sarmat is given upon the nomadic Baghdad, Iraq by five countries namely Iran, Iraq,
Sarmatian tribes which used to live between 6th and Kuwait, Saudi Arabia and Venezuela. OPEC is a
4th century BC in current territories of Russia, permanent, intergovernmental organization and it
Ukraine and Kazakhstan. has a total of 13 Member Countries viz. Iran, Iraq,
Kuwait, United Arab Emirates (UAE), Saudi Arabia,
Algeria, Libya, Nigeria, Gabon, Equatorial Guinea,
Q.99) Ans: d Republic of Congo, Angola and Venezuela are
members of OPEC and its Headquarters in Vienna,
Exp:
Austria. Organization of the Petroleum Exporting
❖ Statement 1 is correct: Wheat is Rabi Crop and the Countries (OPEC) membership is open to any
second most important cereal crop in India after rice. country that is a substantial exporter of oil and
mostly consumed in the north and north-west parts which shares the ideals of the organization.
of the country and grown at a temperature of 10-15
❖ Organization of the Petroleum Exporting Countries
°C (Sowing time) and 21-26 °C (Ripening &
plus (Opec+) refers to the alliance of crude
Harvesting) with bright sunlight. Wheat requires a
producers, who have been undertaking corrections in
rainfall of 75 to 100 cm. . Some states in India are
supply in the oil markets since 2017. OPEC+
Uttar Pradesh, Punjab, Haryana,Madhya Pradesh,
countries include Azerbaijan, Bahrain, Brunei,
Rajasthan, Bihar and Gujarat, the highest producer
Kazakhstan, Malaysia, Mexico, Oman, Russia,
of Wheat.
South Sudan and Sudan
❖ Statement 2 is correct: Ukraine and Russia
accounted for nearly one-third of global wheat
exports. Egypt is the world’s biggest importer of
wheat.

❖ Statement 3 is correct: India is the second-largest


producer of wheat, with a share of around 14.14% of
the world’s total production in 2020. India accounts
for less than 1% of the world’s wheat export. The
top countries importing Indian wheat in 2020-21
were Bangladesh, Nepal, the United Arab Emirates
and Sri Lanka.



PW Web/App: https://smart.link/7wwosivoicgd4
Download more eBooks & Test Series in Hindi and English
from our
Join Telegram- Telegram Channel-
https://t.me/upsc_success_time1
https://t.me/upsc_success_time1

Join Telegram Channel - Click Here

You might also like